*NURSING > STUDY GUIDE > NR 508 / NR508: Advanced Pharmacology Final Exam Q & A (Verified) Test Bank - 2020/2021. A Work, Cha (All)

NR 508 / NR508: Advanced Pharmacology Final Exam Q & A (Verified) Test Bank - 2020/2021. A Work, Chamberlain College of Nursing.

Document Content and Description Below

NR-508 Advanced Pharmacology FINAL EXAM TEST BANK Chapter 1 1. Nurse practitioner prescriptive authority is regulated by: 1. The National Council of State Boards of Nursing 2. The U.S. Drug... Enforcement Administration 3. The State Board of Nursing for each state 4. The State Board of Pharmacy 2. The benefits to the patient of having an Advanced Practice Registered Nurse (APRN) prescriber include: 1. Nurses know more about Pharmacology than other prescribers because they take it both in their basic nursing program and in their APRN program. 2. Nurses care for the patient from a holistic approach and include the patient in decision making regarding their care. 3. APRNs are less likely to prescribe narcotics and other controlled substances. 4. APRNs are able to prescribe independently in all states, whereas a physician’s assistant needs to have a physician supervising their practice. 3. Clinical judgment in prescribing includes: 1. Factoring in the cost to the patient of the medication prescribed 2. Always prescribing the newest medication available for the disease process 3. Handing out drug samples to poor patients 4. Prescribing all generic medications to cut costs 4. Criteria for choosing an effective drug for a disorder include: 1. Asking the patient what drug they think would work best for them 2. Consulting nationally recognized guidelines for disease management 3. Prescribing medications that are available as samples before writing a prescription 4. Following U.S. Drug Enforcement Administration guidelines for prescribing 5. Nurse practitioner practice may thrive under health-care reform because of: 1. The demonstrated ability of nurse practitioners to control costs and improve patient outcomes 2. The fact that nurse practitioners will be able to practice independently 3. The fact that nurse practitioners will have full reimbursement under health-care reform 4. The ability to shift accountability for Medicaid to the state level Chapter 2. Review of Basic Principles of Pharmacology 1. A patient’s nutritional intake and laboratory results reflect hypoalbuminemia. This is critical to prescribing because: 1. Distribution of drugs to target tissue may be affected. 2. The solubility of the drug will not match the site of absorption. 3. There will be less free drug available to generate an effect. 4. Drugs bound to albumin are readily excreted by the kidneys. 2. Drugs that have a significant first-pass effect: 1. Must be given by the enteral (oral) route only 2. Bypass the hepatic circulation 3. Are rapidly metabolized by the liver and may have little if any desired action 4. Are converted by the liver to more active and fat-soluble forms 3. The route of excretion of a volatile drug will likely be the: 1. Kidneys 2. Lungs 3. Bile and feces 4. Skin 4. Medroxyprogesterone (Depo Provera) is prescribed intramuscularly (IM) to create a storage reservoir of the drug. Storage reservoirs: 1. Assure that the drug will reach its intended target tissue 2. Are the reason for giving loading doses 3. Increase the length of time a drug is available and active 4. Are most common in collagen tissues 5. The NP chooses to give cephalexin every 8 hours based on knowledge of the drug’s: 1. Propensity to go to the target receptor 2. Biological half-life 3. Pharmacodynamics 4. Safety and side effects 6. Azithromycin dosing requires that the first day’s dosage be twice those of the other 4 days of the prescription. This is considered a loading dose. A loading dose: 1. Rapidly achieves drug levels in the therapeutic range 2. Requires four- to five-half-lives to attain 3. Is influenced by renal function 4. Is directly related to the drug circulating to the target tissues 7. The point in time on the drug concentration curve that indicates the first sign of a therapeutic effect is the: 1. Minimum adverse effect level 2. Peak of action 3. Onset of action 4. Therapeutic range 8. Phenytoin requires that a trough level be drawn. Peak and trough levels are done: 1. When the drug has a wide therapeutic range 2. When the drug will be administered for a short time only 3. When there is a high correlation between the dose and saturation of receptor sites 4. To determine if a drug is in the therapeutic range 9. A laboratory result indicates that the peak level for a drug is above the minimum toxic concentration. This means that the: 1. Concentration will produce therapeutic effects 2. Concentration will produce an adverse response 3. Time between doses must be shortened 4. Duration of action of the drug is too long 10. Drugs that are receptor agonists may demonstrate what property? 1. Irreversible binding to the drug receptor site 2. Upregulation with chronic use 3. Desensitization or downregulation with continuous use 4. Inverse relationship between drug concentration and drug action 11. Drugs that are receptor antagonists, such as beta blockers, may cause: 1. Downregulation of the drug receptor 2. An exaggerated response if abruptly discontinued 3. Partial blockade of the effects of agonist drugs 4. An exaggerated response to competitive drug agonists 12. Factors that affect gastric drug absorption include: 1. Liver enzyme activity 2. Protein-binding properties of the drug molecule 3. Lipid solubility of the drug 4. Ability to chew and swallow 13. Drugs administered via IV: 1. Need to be lipid soluble in order to be easily absorbed 2. Begin distribution into the body immediately 3. Are easily absorbed if they are nonionized 4. May use pinocytosis to be absorbed 14. When a medication is added to a regimen for a synergistic effect, the combined effect of the drugs is: 1. The sum of the effects of each drug individually 2. Greater than the sum of the effects of each drug individually 3. Less than the effect of each drug individually 4. Not predictable, as it varies with each individual 15. Which of the following statements about bioavailability is true? 1. Bioavailability issues are especially important for drugs with narrow therapeutic ranges or sustained-release mechanisms. 2. All brands of a drug have the same bioavailability. 3. Drugs that are administered more than once a day have greater bioavailability than drugs given once daily. 4. Combining an active drug with an inert substance does not affect bioavailability. 16. Which of the following statements about the major distribution barriers (blood-brain or fetal-placental) is true? 1. Water soluble and ionized drugs cross these barriers rapidly. 2. The blood-brain barrier slows the entry of many drugs into and from brain cells. 3. The fetal-placental barrier protects the fetus from drugs taken by the mother. 4. Lipid-soluble drugs do not pass these barriers and are safe for pregnant women. 17. Drugs are metabolized mainly by the liver via phase I or phase II reactions. The purpose of both of these types of reactions is to: 1. Inactivate prodrugs before they can be activated by target tissues 2. Change the drugs so they can cross plasma membranes 3. Change drug molecules to a form that an excretory organ can excrete 4. Make these drugs more ionized and polar to facilitate excretion 18. Once they have been metabolized by the liver, the metabolites may be: 1. More active than the parent drug 2. Less active than the parent drug 3. Totally “deactivated” so they are excreted without any effect 4. All of the above 19. All drugs continue to act in the body until they are changed or excreted. The ability of the body to excrete drugs via the renal system would be increased by: 1. Reduced circulation and perfusion of the kidney 2. Chronic renal disease 3. Competition for a transport site by another drug 4. Unbinding a nonvolatile drug from plasma proteins 20. Steady state is: 1. The point on the drug concentration curve when absorption exceeds excretion 2. When the amount of drug in the body remains constant 3. When the amount of drug in the body stays below the minimum toxic concentration 4. All of the above 21. Two different pain medications are given together for pain relief. The drug—drug interaction is: 1. Synergistic 2. Antagonistic 3. Potentiative 4. Additive 22. Actions taken to reduce drug—drug interaction problems include all of the following EXCEPT: 1. Reducing the dosage of one of the drugs 2. Scheduling their administration at different times 3. Prescribing a third drug to counteract the adverse reaction of the combination 4. Reducing the dosage of both drugs 23. Phase I oxidative-reductive processes of drug metabolism require certain nutritional elements. Which of the following would reduce or inhibit this process? 1. Protein malnutrition 2. Iron-deficiency anemia 3. Both 1 and 2 4. Neither 1 nor 2 24. The time required for the amount of drug in the body to decrease by 50% is called: 1. Steady state 2. Half-life 3. Phase II metabolism 4. Reduced bioavailability time 25. An agonist activates a receptor and stimulates a response. When given frequently over time, the body may: 1. Upregulate the total number of receptors 2. Block the receptor with a partial agonist 3. Alter the drug’s metabolism 4. Downregulate the numbers of that specific receptor 26. Drug antagonism is best defined as an effect of a drug that: 1. Leads to major physiological and psychological dependence 2. Is modified by the concurrent administration of another drug 3. Cannot be metabolized before another dose is administered 4. Leads to a decreased physiological response when combined with another drug 27. Instructions to a client regarding self-administration of oral enteric-coated tablets should include which of the following statements? 1. “Avoid any other oral medicines while taking this drug.” 2. “If swallowing this tablet is difficult, dissolve it in 3 ounces of orange juice.” 3. “The tablet may be crushed if you have any difficulty taking it.” 4. “To achieve best effect, take the tablet with at least 8 ounces of fluid.” 28. The major reason for not crushing a sustained-release capsule is that, if crushed, the coated beads of the drugs could possibly result in: 1. Disintegration 2. Toxicity 3. Malabsorption 4. Deterioration 29. Which of the following substances is the most likely to be absorbed in the intestines rather than in the stomach? 1. Sodium bicarbonate 2. Ascorbic acid 3. Salicylic acid 4. Glucose 30. Which of the following variables is a factor in drug absorption? 1. The smaller the surface area for absorption, the more rapidly the drug is absorbed. 2. A rich blood supply to the area of absorption leads to better absorption. 3. The less soluble the drug, the more easily it is absorbed. 4. Ionized drugs are easily absorbed across the cell membrane. 31. An advantage of prescribing a sublingual medication is that the medication is: 1. Absorbed rapidly 2. Excreted rapidly 3. Metabolized minimally 4. Distributed equally 32. Drugs that use CYP 3A4 isoenzymes for metabolism may: 1. Induce the metabolism of another drug 2. Inhibit the metabolism of another drug 3. Both 1 and 2 4. Neither 1 nor 2 33. Therapeutic drug levels are drawn when a drug reaches steady state. Drugs reach steady state: 1. After the second dose 2. After four to five half-lives 3. When the patient feels the full effect of the drug 4. One hour after IV administration 34. Upregulation or hypersensitization may lead to: 1. Increased response to a drug 2. Decreased response to a drug 3. An exaggerated response if the drug is withdrawn 4. Refractoriness or complete lack of response Chapter 3. Rational Drug Selection 1. An NP would prescribe the liquid form of ibuprofen for a 6-year-old child because: 1. Drugs given in liquid form are less irritating to the stomach. 2. A 6-year-old child may have problems swallowing a pill. 3. Liquid forms of medication eliminate the concern for first-pass effect. 4. Liquid ibuprofen does not have to be dosed as often as the tablet form. 2. In deciding which of multiple drugs used to use to treat a condition, the NP chooses Drug A because it: 1. Has serious side effects and it is not being used for a life-threatening condition 2. Will be taken twice daily and will be taken at home 3. Is expensive, but covered by health insurance 4. None of these are important in choosing a drug 3. A client asks the NP about the differences in drug effects between men and women. What is known about the differences between the pharmacokinetics of men and women? 1. Body temperature varies between men and women. 2. Muscle mass is greater in women. 3. Percentage of fat differs between genders. 4. Proven subjective factors exist between the genders. 4. The first step in the prescribing process according to the World Health Organization is: 1. Choosing the treatment 2. Educating the patient about the medication 3. Diagnosing the patient’s problem 4. Starting the treatment 5. Treatment goals in prescribing should: 1. Always be curative 2. Be patient-centered 3. Be convenient for the provider 4. Focus on the cost of therapy 6. The therapeutic goals when prescribing include(s): 1. Curative 2. Palliative 3. Preventive 4. All of the above 7. When determining drug treatment the NP prescriber should: 1. Always use evidence-based guidelines 2. Individualize the drug choice for the specific patient 3. Rely on his or her experience when prescribing for complex patients 4. Use the newest drug on the market for the condition being treated 8. Patient education regarding prescribed medication includes: 1. Instructions written at the high school reading level 2. Discussion of expected adverse drug reactions 3. How to store leftover medication such as antibiotics 4. Verbal instructions always in English 9. Passive monitoring of drug effectiveness includes: 1. Therapeutic drug levels 2. Adding or subtracting medications from the treatment regimen 3. Ongoing provider visits 4. Instructing the patient to report if the drug is not effective 10. Pharmacokinetic factors that affect prescribing include: 1. Therapeutic index 2. Minimum effective concentration 3. Bioavailability 4. Ease of titration 11. Pharmaceutical promotion may affect prescribing. To address the impact of pharmaceutical promotion, the following recommendations have been made by the Institute of Medicine: 1. Conflicts of interest and financial relationships should be disclosed by those providing education. 2. Providers should ban all pharmaceutical representatives from their office setting. 3. Drug samples should be used for patients who have the insurance to pay for them, to ensure the patient can afford the medication. 4. Providers should only accept low-value gifts, such as pens and pads of paper, from the pharmaceutical representative. 12. Under new U.S. Food and Drug Administration labeling, Pregnancy Categories will be: 1. Strengthened with a new coding such as C+ or C- to discern when a drug is more or less toxic to the fetus 2. Changed to incorporate a pregnancy risk summary and clinical considerations on the drug label 3. Eliminated, and replaced with a link to the National Library of Medicine TOXNET Web site for in-depth information regarding pregnancy concerns 4. Clarified to include information such as safe dosages in each trimester of pregnancy Chapter 4. Legal and Professional Issues in Prescribing 1. The U.S. Food and Drug Administration regulates: 1. Prescribing of drugs by MDs and NPs 2. The official labeling for all prescription and over-the-counter drugs 3. Off-label recommendations for prescribing 4. Pharmaceutical educational offerings 2. The U.S. Food and Drug Administration approval is required for: 1. Medical devices, including artificial joints 2. Over-the-counter vitamins 3. Herbal products, such as St John’s wort 4. Dietary supplements, such as Ensure 3. An Investigational New Drug is filed with the U.S. Food and Drug Administration: 1. When the manufacturer has completed phase III trials 2. When a new drug is discovered 3. Prior to animal testing of any new drug entity 4. Prior to human testing of any new drug entity 4. Phase IV clinical trials in the United States are also known as: 1. Human bioavailability trials 2. Postmarketing research 3. Human safety and efficacy studies 4. The last stage of animal trials before the human trials begin 5. Off-label prescribing is: 1. Regulated by the U.S. Food and Drug Administration 2. Illegal by NPs in all states (provinces) 3. Legal if there is scientific evidence for the use 4. Regulated by the Drug Enforcement Administration 6. The U.S. Drug Enforcement Administration: 1. Registers manufacturers and prescribers of controlled substances 2. Regulates NP prescribing at the state level 3. Sanctions providers who prescribe drugs off-label 4. Provides prescribers with a number they can use for insurance billing 7. Drugs that are designated Schedule II by the U.S. Drug Enforcement Administration: 1. Are known teratogens during pregnancy 2. May not be refilled; a new prescription must be written 3. Have a low abuse potential 4. May be dispensed without a prescription unless regulated by the state 8. Precautions that should be taken when prescribing controlled substances include: 1. Faxing the prescription for a Schedule II drug directly to the pharmacy 2. Using tamper-proof paper for all prescriptions written for controlled drugs 3. Keeping any pre-signed prescription pads in a locked drawer in the clinic 4. Using only numbers to indicate the amount of drug to be prescribed 9. Strategies prescribers can use to prevent misuse of controlled prescription drugs include: 1. Use of chemical dependency screening tools 2. Firm limit-setting regarding prescribing controlled substances 3. Practicing “just say no” to deal with patients who are pushing the provider to prescribe controlled substances 4. All of the above 10. Behaviors predictive of addiction to controlled substances include: 1. Stealing or borrowing another patient’s drugs 2. Requiring increasing doses of opiates for pain associated with malignancy 3. Receiving refills of a Schedule II prescription on a regular basis 4. Requesting that only their own primary care provider prescribe for them 11. Medication agreements or “Pain Medication Contracts” are recommended to be used: 1. Universally for all prescribing for chronic pain 2. For patients who have repeated requests for pain medication 3. When you suspect a patient is exhibiting drug-seeking behavior 4. For patients with pain associated with malignancy 12. A prescription needs to be written for: 1. Legend drugs 2. Most controlled drugs 3. Medical devices 4. All of the above Chapter 5. Adverse Drug Reactions 1. Which of the following patients would be at higher risk of experiencing adverse drug reactions (ADRs): 1. A 32-year-old male 2. A 22-year-old female 3. A 3-month-old female 4. A 48-year-old male 2. Infants and young children are at higher risk of ADRs due to: 1. Immature renal function in school-age children 2. Lack of safety and efficacy studies in the pediatric population 3. Children’s skin being thicker than adults, requiring higher dosages of topical medication 4. Infant boys having a higher proportion of muscle mass, leading to a higher volume of distribution 3. The elderly are at high risk of ADRs due to: 1. Having greater muscle mass than younger adults, leading to higher volume of distribution 2. The extensive studies that have been conducted on drug safety in this age group 3. The blood-brain barrier being less permeable, requiring higher doses to achieve therapeutic effect 4. Age-related decrease in renal function 4. The type of adverse drug reaction that is idiosyncratic when a drug given in the usual therapeutic doses is type: 1. A 2. B 3. C 4. D 5. Digoxin may cause a type A adverse drug reaction due to: 1. Idiosyncratic effects 2. Its narrow therapeutic index 3. Being a teratogen 4. Being a carcinogen 6. Sarah developed a rash after using a topical medication. This is a type __ allergic drug reaction. 1. I 2. II 3. III 4. IV 7. A patient may develop neutropenia from using topical Silvadene for burns. Neutropenia is a(n): 1. Cytotoxic hypersensitivity reaction 2. Immune complex hypersensitivity 3. Immediate hypersensitivity reaction 4. Delayed hypersensitivity reaction 8. Anaphylactic shock is a: 1. Type I reaction, called immediate hypersensitivity reaction 2. Type II reaction, called cytotoxic hypersensitivity reaction 3. Type III allergic reaction, called immune complex hypersensitivity 4. Type IV allergic reaction, called delayed hypersensitivity reaction 9. James has hypothalamic-pituitary-adrenal axis suppression from chronic prednisone (a corticosteroid) use. He is at risk for what type of adverse drug reaction? 1. Type B 2. Type C 3. Type E 4. Type F 10. Immunomodulators such as azathioprine may cause a delayed adverse drug reaction known as a type D reaction because they are known: 1. Teratogens 2. Carcinogens 3. To cause hypersensitivity reactions 4. Hypothalamus-pituitary-adrenal axis suppressants 11. A 24-year-old male received multiple fractures in a motor vehicle accident that required significant amounts of opioid medication to treat his pain. He is at risk for a _____ adverse drug reaction when he no longer requires the opioids. 1. Rapid 2. First-dose 3. Late 4. Delayed 12. An example of a first-dose reaction that may occur includes: 1. Orthostatic hypotension that does not occur with repeated doses 2. Purple glove syndrome with phenytoin use 3. Hemolytic anemia from ceftriaxone use 4. Contact dermatitis from neomycin use 13. Drugs that are prone to cause adverse drug effects include: 1. Diuretics 2. Inhaled anticholinergics 3. Insulins 4. Stimulants 14. The U.S. Food and Drug Administration MedWatch system is activated when: 1. There is an adverse event to a vaccine. 2. The patient has a severe reaction that is noted in the “Severe Reaction” section in the medication label. 3. A lactating woman takes a medication that is potentially toxic to the breastfeeding infant. 4. An adverse event or serious problem occurs with a medication that is not already identified on the label. 15. The Vaccine Adverse Events Reporting System is: 1. A mandatory reporting system for all health-care providers when they encounter an adverse vaccine event 2. A voluntary reporting system that health-care providers or consumers may use to report vaccine adverse events 3. Utilized to send out safety alerts regarding emerging vaccine safety issues 4. Activated when a vaccine has been proven to cause significant adverse effects Chapter 6. Factors That Foster Positive Outcomes 1. A comprehensive assessment of a patient should be holistic when trying to determine competence in drug administration. Which of the following factors would the NP omit from this type of assessment? 1. Financial status 2. Mobility 3. Social support 4. Sexual practices 2. Elena Vasquez’s primary language is Spanish, and she speaks very limited English. Which technique would be appropriate to use in teaching her about a new drug you have just prescribed? 1. Use correct medical terminology because Spanish has a Latin base. 2. Use a family member who speaks more English to act as an interpreter. 3. Use a professional interpreter or a reliable staff member who can act as an interpreter. 4. Use careful, detailed explanations. 3. Rod, age 68, has hearing difficulty. Which of the following would NOT be helpful in assuring that he understands teaching about his drug? 1. Stand facing him and speak slowly and clearly. 2. Speak in low tones or find a provider who has a lower voice. 3. Write down the instructions as well as speaking them. 4. If he reads lips, exaggerate lips movements when pronouncing the vowel sounds. 4. Which of the following factors may adversely affect a patient’s adherence to a therapeutic drug regimen? 1. Complexity of the drug regimen 2. Patient perception of the potential adverse effects of the drugs 3. Both 1 and 2 4. Neither 1 nor 2 5. The health-care delivery system itself can create barriers to adherence to a treatment regimen. Which of the following system variables creates such a barrier? 1. Increasing copayments for care 2. Unrestricted formularies for drugs, including brand names 3. Increasing the number of people who have access to care 4. Treating a wider range of disorders 6. Ralph’s blood pressure remains elevated despite increased doses of his drug. The NP is concerned that he might not be adhering to his treatment regimen. Which of the following events would suggest that he might not be adherent? 1. Ralph states that he always takes the drug “when I feel my pressure is going up.” 2. Ralph contacts his NP to discuss the need to increase the dosage. 3. Ralph consistently keeps his follow-up appointments to check his blood pressure. 4. All of the above show that he is adherent to the drug regimen. 7. Nonadherence is especially common in drugs that treat asymptomatic conditions, such as hypertension. One way to reduce the likelihood of nonadherence to these drugs is to prescribe a drug that: 1. Has a short half-life so that missing one dose has limited effect 2. Requires several dosage titrations so that missed doses can be replaced with lower doses to keep costs down 3. Has a tolerability profile with fewer of the adverse effects that are considered “irritating,” such as nausea and dizziness 4. Must be taken no more than twice a day 8. Factors in chronic conditions that contribute to nonadherence include: 1. The complexity of the treatment regimen 2. The length of time over which it must be taken 3. Breaks in the usual daily routine, such as vacations and weekends 4. All of the above 9. While patient education about their drugs is important, information alone does not necessarily lead to adherence to a drug regimen. Patients report greater adherence when: 1. The provider spent a lot of time discussing the drugs with them 2. Their concerns and specific area of knowledge deficit were addressed 3. They were given written material, such as pamphlets, about the drugs 4. The provider used appropriate medical and pharmacological terms 10. Patients with psychiatric illnesses have adherence rates to their drug regimen between 35% and 60%. To improve adherence in this population, prescribe drugs: 1. With a longer half-life so that missed doses produce a longer taper on the drug curve 2. In oral formulations that are more easily taken 3. That do not require frequent monitoring 4. Combined with patient education about the need to adhere even when symptoms are absent 11. Many disorders require multiple drugs to treat them. The more complex the drug regimen, the less likely the patient will adhere to it. Which of the following interventions will NOT improve adherence? 1. Have the patient purchase a pill container with compartments for daily or multiple times-per-day dosing. 2. Match the clinic appointment to the next time the drug is to be refilled. 3. Write prescriptions for new drugs with shorter times between refills. 4. Give the patient a clear drug schedule that the provider devises to fit the characteristic of the drug. 12. Pharmacologic interventions are costly. Patients for whom the cost/benefit variable is especially important include: 1. Older adults and those on fixed incomes 2. Patients with chronic illnesses 3. Patients with copayments for drugs on their insurance 4. Patients on public assistance 13. Providers have a responsibility for determining the best plan of care, but patients also have responsibilities. Patients the provider can be assured will carry through on these responsibilities include those who: 1. Are well-educated and affluent 2. Have chronic conditions 3. Self-monitor drug effects on their symptoms 4. None of the above guarantee adherence 14. Monitoring adherence can take several forms, including: 1. Patient reports from data in a drug diary 2. Pill counts 3. Laboratory reports and other diagnostic markers 4. All of the above 15. Factors that explain and predict medication adherence include: 1. Social 2. Financial 3. Health system 4. All of the above Chapter 9. Nutrition and Nutraceuticals 1. The most frequent type of drug-food interaction is food: 1. Causing increased therapeutic drug levels 2. Affecting the metabolism of the drug 3. Altering the volume of distribution of drugs 4. Affecting the gastrointestinal absorption of drugs 2. Food in the gastrointestinal tract affects drug absorption by: 1. Altering the pH of the colon, which decreases absorption 2. Competing with the drug for plasma proteins 3. Altering gastric emptying time 4. Altering the pH of urine 3. Food can alter the pH of the stomach, leading to: 1. Enhanced drug metabolism 2. Altered vitamin K absorption 3. Increased vitamin D absorption 4. Altered drug bioavailability 4. Fasting for an extended period can: 1. Increase drug absorption due to lack of competition between food and the drug 2. Alter the pH of the gastrointestinal tract, affecting absorption 3. Cause vasoconstriction, leading to decreased drug absorption 4. Shrink the stomach, causing decreased surface area for drug absorption 5. Tetracycline needs to be given on an empty stomach because it chelates with: 1. Calcium 2. Magnesium 3. Iron 4. All of the above 6. A low-carbohydrate, high-protein diet may: 1. Increase drug-metabolizing enzymes 2. Decrease drug absorption from the GI tract 3. Alter drug binding to plasma proteins 4. Enhance drug elimination 7. Grapefruit juice contains furanocoumarins that have been found to: 1. Alter absorption of drugs through competition for binding sites 2. Inhibit CYP 3A4, leading to decreased first-pass metabolism of drugs 3. Alter vitamin K metabolism, leading to prolonged bleeding 4. Enhance absorption of calcium and vitamin D 8. Cruciferous vegetables may alter drug pharmacokinetics by: 1. Enhancing absorption of weakly acidic drugs 2. Altering CYP 3A4 activity, leading to elevated levels of drugs, such as the statins 3. Inducing CYP 1A2, possibly leading to therapeutic failure of drugs metabolized by CYP 1A2 4. Decreasing first-pass metabolism of drugs 9. Milk and other foods that alkalinize the urine may: 1. Result in basic drugs being reabsorbed in the renal tubule 2. Increase the elimination of basic drugs in the urine 3. Decrease the elimination of acidic drugs 4. Not alter drug elimination due to the minimal change in urine pH 10. Antacids such as calcium carbonate (Tums) can reduce the absorption of which of the following nutrients? 1. Protein 2. Calcium 3. Iron 4. Vitamin K 11. Phenytoin decreases folic acid absorption by: 1. Altering the pH of the stomach 2. Increasing gastric emptying time 3. Inhibiting intestinal enzymes required for folic acid absorption 4. Chelation of the folic acid into inactive ingredients 12. Patients taking warfarin need to be educated about the vitamin K content of foods to avoid therapeutic failure. Foods high in vitamin K that should be limited to no more than one serving per day include: 1. Spinach 2. Milk 3. Romaine lettuce 4. Cauliflower 13. The American Dietetic Association has recommended the use of specific nutritional supplements in the following population(s): 1. 400 IU per day of vitamin D in all infants and children 2. 1,000 IU per day of vitamin D for all pregnant women 3. 60 mg per day of iron for all adults over age 50 years 4. All of the above 14. The American Dietetic Association recommends pregnant women take a supplement including: 1. 1,000 IU daily of vitamin D 2. 2.4 mcg/day of vitamin B12 3. 600 mcg/day of folic acid 4. 8 mg/day of iron 15. The American Heart Association and the American Dietetic Association recommend a minimum daily fiber intake of ______ for cardiovascular health: 1. 10 mg/day 2. 15 mg/day 3. 20 mg/day 4. 25 mg/day 16. Which of the following vitamin or mineral supplements may by teratogenic if a pregnant woman takes more than the recommended amount? 1. Iron 2. Vitamin A 3. Vitamin B6 4. Vitamin C 17. Vitamin B2 (riboflavin) may be prescribed to: 1. Decrease the incidence of beriberi 2. Reduce headaches and migraines 3. Prevent pernicious anemia 4. Treat hyperlipidemia 18. Isoniazid (INH) may induce a deficiency of which vitamin? 1. B6 2. C 3. D 4. E 19. Pregnant patients who are taking isoniazid (INH) should take 25 mg/day of vitamin B6 (pyridoxine) to prevent: 1. Beriberi 2. Peripheral neuropathy 3. Rickets 4. Megaloblastic anemia 20. Vitamin B12 deficiency may lead to: 1. Hair loss 2. Insomnia 3. Dry scales on the scalp 4. Numbness and tingling of the hands 21. Smokers are at risk for vitamin C deficiency. It is recommended that smokers take _______ vitamin C supplement. 1. 100 mg/day 2. 500 mg/day 3. 1,000 mg/day 4. 35 mg/day more than nonsmokers 22. There is strong evidence to support that adequate vitamin C intake prevents: 1. The common cold 2. Breast cancer 3. Scurvy 4. All of the above 23. Adequate vitamin D is needed for: 1. Absorption of calcium from the gastrointestinal tract 2. Regulation of serum calcium levels 3. Regulation of serum phosphate levels 4. All of the above 24. Newborns are at risk for early vitamin K deficiency bleeding and the American Academy of Pediatrics recommends that all newborns receive: 1. IM vitamin K (phytonadione) within 24 hours of birth 2. Oral vitamin K supplementation in the first 3 weeks of life 3. Formula containing vitamin K or breast milk 4. Oral vitamin K in the first 24 hours after birth 25. Symptoms of folate deficiency include: 1. Thinning of the hair 2. Bruising easily 3. Glossitis 4. Numbness and tingling of the hands and feet 26. A patient with a new onset of systolic ejection murmur should be assessed for which nutritional deficiency? 1. Vitamin B12 2. Vitamin C 3. Folate 4. Niacine 27. According to the 2003-2006 National Health and Nutrition Examination Survey study of dietary intake, the group at highest risk for inadequate calcium intake was: 1. The elderly (over age 60 years) 2. Teenage females 3. Teenage males 4. Preschoolers 28. Patients with iron deficiency will develop: 1. Hemolytic anemia 2. Megaloblastic anemia 3. Macrocytic-hypochromic anemia 4. Microcytic-hypochromic anemia 29. There is evidence that dietary supplementation or adequate intake of fish oils and omega-3 fatty acids have well-documented: 1. Concern for developing cardiac dysrhythmias 2. Anti-inflammatory effects 3. Total cholesterol-lowering effects 4. Effects on fasting blood sugar 30. There is enough preliminary evidence to recommend that children with autism receive which supplemental nutrient? 1. Vitamin B1 (thiamine) 2. Vitamin B2 (riboflavin) 3. Calcium 4. Omega-3 fatty acids 31. There is sufficient evidence to support the use of omega-3 fatty acids to treat the following disease(s): 1. Asthma 2. Autism 3. Arthritis 4. All of the above 32. It is reasonable to recommend supplementation with _________ in the treatment of hyperlipidemia. 1. Omega-3 fatty acids 2. Probiotics 3. Plant sterols 4. Calcium 33. Probiotics are recommended to be co-administered when ____________ are prescribed: 1. Antacids 2. Antihypertensives 3. Antidiarrheals 4. Antibiotics 34. It is reasonable to add ___________ to a Helicobacter pylori treatment regimen to improve eradication rates of H. pylori. 1. Probiotics 2. Omega-3 fatty acids 3. Plant sterols 4. Fiber Chapter 13. Over-the-Counter Medications 1. Michael asks you about why some drugs are over-the-counter and some are prescription. You explain that in order for a drug to be approved for over-the-counter use the drug must: 1. Be safe and labeled for appropriate use 2. Have a low potential for abuse or misuse 3. Be taken for a condition the patient can reliably self-diagnose 4. All of the above 2. In the United States, over-the-counter drugs are regulated by: 1. No one. There is no oversight for over-the-counter medications. 2. The U.S. Food and Drug Administration Center for Drug Evaluation and Research 3. The U.S. Drug Enforcement Administration 4. MedWatch 3. As drugs near the end of their patent, pharmaceutical companies may apply for the drug to change to over-the-counter status in order to: 1. Get a new patent for the over-the-counter form of the drug 2. Lower the costs because most prescription benefit plans do not cover generics 3. Market the drug to a whole new population, as they are able to market to patients instead of just providers 4. Continue to make large profits from their blockbuster brand-name drug 4. New over-the-counter drug ingredients must undergo the U.S. Food and Drug Administration New Drug Application process, just as prescription drugs do. 1. True 2. False 5. The ailment that generates the greatest over-the-counter annual drug sales is: 1. Constipation 2. Cough and colds 3. Heartburn 4. Acute and chronic pain 6. Common over-the-counter pain relievers such as acetaminophen or ibuprofen: 1. Are always safer for the patient than prescription pain medication 2. Are harmful if taken in higher than recommended amounts 3. Have minimal interaction with prescription medications 4. Should never be given to children unless recommended by their provider 7. When obtaining a drug history from Harold, he gives you a complete list of his prescription medications. He denies taking any other drugs, but you find that he occasionally takes aspirin for his arthritis flare ups. This is an example of: 1. His appropriately only telling you about his regularly prescribed medications 2. His hiding information regarding his inappropriate use of aspirin from you 3. A common misconception that intermittently taken over-the counter medications are not an important part of his drug history 4. A common misuse of over-the-counter aspirin 8. The Combat Methamphetamine Epidemic Act, which is part of the 2006 U.S. Patriot Act: 1. Requires all providers to screen their patients for methamphetamine use 2. Restricts the prescribing of amphetamines to U.S. citizens 3. Requires a prescription be written for all methamphetamine precursors in all states 4. Restricts the sales of drugs that contain methamphetamine precursors, including a daily and 30-day limit on sales 9. When prescribing a tetracycline or quinolone antibiotic it is critical to instruct the patient: 1. Not to take their regularly prescribed medications while on these antibiotics 2. Regarding the need for lots of acidic foods and juices, such as orange juice, to enhance absorption 3. Not to take antacids while on these medications, as the antacid decreases absorption 4. That there are no drug interactions with these antibiotics chapter 14. Drugs Affecting the Autonomic Nervous System ok 1. Charlie is a 65-year-old male who has been diagnosed with hypertension and benign prostatic hyperplasia. Doxazosin has been chosen to treat his hypertension because it: 1. Increases peripheral vasoconstriction 2. Decreases detrusor muscle contractility 3. Lowers supine blood pressure more than standing pressure 4. Relaxes smooth muscle in the bladder neck 2. To reduce potential adverse effects, patients taking a peripherally acting alpha1 antagonist should do all of the following EXCEPT: 1. Take the dose at bedtime 2. Sit up slowly and dangle their feet before standing 3. Monitor their blood pressure and skip a dose if the pressure is less than 120/80 4. Weigh daily and report weight gain of greater than 2 pounds in one day 3. John has clonidine, a centrally acting adrenergic blocker, prescribed for his hypertension. He should: 1. Not miss a dose or stop taking the drug because of potential rebound hypertension 2. Increase fiber in the diet to treat any diarrhea that may occur 3. Reduce fluid intake to less than 2 liters per day to prevent fluid retention 4. Avoid sitting for long periods, as this can lead to deep vein thrombosis 4. Clonidine has several off-label uses, including: 1. Alcohol and nicotine withdrawal 2. Post-herpetic neuralgia 3. Both 1 and 2 4. Neither 1 nor 2 5. Jim is being treated for hypertension. Because he has a history of heart attack, the drug chosen is atenolol. Beta blockers treat hypertension by: 1. Increasing heart rate to improve cardiac output 2. Reducing vascular smooth muscle tone 3. Increasing aldosterone-mediated volume activity 4. Reducing aqueous humor production 6. Which of the following adverse effects are less likely in a beta1-selective blocker? 1. Dysrhythmias 2. Impaired insulin release 3. Reflex orthostatic changes 4. Decreased triglycerides and cholesterol 7. Richard is 70 years old and has a history of cardiac dysrhythmias. He has been prescribed nadolol. You do his annual laboratory work and find a CrCl of 25 ml/min. What action should you take related to his nadolol? 1. Extend the dosage interval. 2. Decrease the dose by 75%. 3. Take no action because this value is expected in the older adult. 4. Schedule a serum creatinine level to validate the CrCl value. 8. Beta blockers are the drugs of choice for exertional angina because they: 1. Improve myocardial oxygen supply by vasodilating the coronary arteries 2. Decrease myocardial oxygen demand by decreasing heart rate and vascular resistance 3. Both 1 and 2 4. Neither 1 nor 2 9. Adherence to beta blocker therapy may be affected by their: 1. Short half-lives requiring twice daily dosing 2. Tendency to elevate lipid levels 3. Effects on the male genitalia, which may produce impotence 4. None of the above 10. Beta blockers have favorable effects on survival and disease progression in heart failure. Treatment should be initiated when the: 1. Symptoms are severe 2. Patient has not responded to other therapies 3. Patient has concurrent hypertension 4. Left ventricular dysfunction is diagnosed 11. Abrupt withdrawal of beta blockers can be life threatening. Patients at highest risk for serious consequences of rapid withdrawal are those with: 1. Angina 2. Coronary artery disease 3. Both 1 and 2 4. Neither 1 nor 2 12. To prevent life-threatening events from rapid withdrawal of a beta blocker: 1. The dosage interval should be increased by 1 hour each day. 2. An alpha blocker should be added to the treatment regimen before withdrawal. 3. The dosage should be tapered over a period of weeks. 4. The dosage should be decreased by one-half every 4 days. 13. Beta blockers are prescribed for diabetics with caution because of their ability to produce hypoglycemia and block the common symptoms of it. Which of the following symptoms of hypoglycemia is not blocked by these drugs and so can be used to warn diabetics of possible decreased blood glucose? 1. Dizziness 2. Increased heart rate 3. Nervousness and shakiness 4. Diaphoresis 14. Combined alpha-beta antagonists are used to reduce the progression of heart failure because they: 1. Vasodilate the peripheral vasculature 2. Decrease cardiac output 3. Increase renal vascular resistance 4. Reduce atherosclerosis secondary to elevated serum lipoproteins 15. Carvedilol is heavily metabolized by CYP2D6 and 2C9, resulting in drug interactions with which of the following drug classes? 1. Histamine 2 blockers 2. Quinolones 3. Serotonin re-uptake inhibitors 4. All of the above 16. Alpha-beta blockers are especially effective to treat hypertension for which ethnic group? 1. White 2. Asian 3. African American 4. Native American 17. Bethanechol: 1. Increases detrusor muscle tone to empty the bladder 2. Decreases gastric acid secretion to treat peptic ulcer disease 3. Stimulates voluntary muscle tone to improve strength 4. Reduces bronchial airway constriction to treat asthma 18. Clinical dosing of Bethanechol: 1. Begins at the highest effective dose to obtain a rapid response 2. Starts at 5 mg to 10 mg PO and is repeated every hour until a satisfactory clinical response is achieved 3. Requires dosing only once daily 4. Is the same for both the oral and parenteral route 19. Patients who need to remain alert are taught to avoid which drug due to its antimuscarinic effects? 1. Levothyroxine 2. Prilosec 3. Dulcolax 4. Diphenhydramine 20. Anticholinesterase inhibitors are used to treat: 1. Peptic ulcer disease 2. Myasthenia gravis 3. Both 1 and 2 4. Neither 1 nor 2 21. Which of the following drugs used to treat Alzheimer’s disease is not an anticholinergic? 1. Donepezil 2. Memantine 3. Rivastigmine 4. Galantamine 22. Taking which drug with food maximizes it bioavailability? 1. Donepezil 2. Galantamine 3. Rivastigmine 4. Memantine 23. Which of the following drugs should be used only when clearly needed in pregnant and breastfeeding women? 1. Memantine 2. Pyridostigmine 3. Galantamine 4. Rivastigmine 24. There is a narrow margin between first appearance of adverse reaction to AChE inhibitors and serious toxic effects. Adverse reactions that require immediate action include: 1. Dizziness and headache 2. Nausea 3. Decreased salivation 4. Fasciculations of voluntary muscles 25. Adherence is improved when a drug can be given once daily. Which of the following drugs can be given once daily? 1. Tacrine 2. Donepezil 3. Memantine 4. Pyridostigmine 26. Nicotine has a variety of effects on nicotinic receptors throughout the body. Which of the following is NOT an effect of nicotine? 1. Vasodilation and decreased heart rate 2. Increased secretion of gastric acid and motility of the GI smooth muscle 3. Release of dopamine at the pleasure center 4. Stimulation of the locus coeruleus 27. Nicotine gum products are: 1. Chewed to release the nicotine and then swallowed for a systemic effect 2. “Parked” in the buccal area of the mouth to produce a constant amount of nicotine release 3. Bound to exchange resins so the nicotine is only released during chewing 4. Approximately the same in nicotine content as smoking two cigarettes 28. Nicotine replacement therapy (NRT): 1. Is widely distributed in the body only when the gum products are used 2. Does not cross the placenta and so is safe for pregnant women 3. Delays healing of esophagitis and peptic ulcers 4. Has no drug interactions when a transdermal patch is used 29. Success rates for smoking cessation using NRT: 1. Are about the same regardless of the method chosen 2. Vary from 40% to 50% at 12 months 3. Both 1 and 2 4. Neither 1 nor 2 30. Cholinergic blockers are used to: 1. Counteract the extrapyramidal symptoms (EPS) effects of phenothiazines 2. Control tremors and relax smooth muscle in Parkinson’s disease 3. Inhibit the muscarinic action of ACh on bladder muscle 4. All of the above 31. Several classes of drugs have interactions with cholinergic blockers. Which of the following is true about these interactions? 1. Drugs with a narrow therapeutic range given orally may not stay in the GI tract long enough to produce an action. 2. Additive antimuscarinic effects may occur with antihistamines. 3. Cholinergic blockers may decrease the sedative effects of hypnotics. 4. Cholinergic blockers are contraindicated with antipsychotics. 32. Scopolamine can be used to prevent the nausea and vomiting associated with motion sickness. The patient is taught to: 1. Apply the transdermal disk at least 4 hours before the antiemetic effect is desired. 2. Swallow the tablet 1 hour before traveling where motion sickness is possible. 3. Place the tablet under the tongue and allow it to dissolve. 4. Change the transdermal disk daily for maximal effect. 33. You are managing the care of a patient recently diagnosed with benign prostatic hyperplasia (BPH). He is taking tamsulosin but reports dizziness when standing abruptly. The best option for this patient is: 1. Continue the tamsulosin because the side effect will resolve with continued treatment. 2. Discontinue the tamsulosin and start doxazosin. 3. Have him double his fluid intake and stand more slowly. 4. Prescribe meclizine as needed for the dizziness. 34. You are treating a patient with a diagnosis of Alzheimer’s disease. The patient’s wife mentions difficulty with transportation to the clinic. Which medication is the best choice? 1. Donepezil 2. Tacrine 3. Doxazosin 4. Verapamil 35. A patient presents with a complaint of dark stools and epigastric pain described as gnawing and burning. Which of the medications is the most likely cause? 1. Acetaminophen 2. Estradiol 3. Donepezil 4. Bethanechol 36. Your patient calls for an appointment before going on vacation. Which medication should you ensure he has an adequate supply of before leaving to avoid life-threatening complications? 1. Carvedilol 2. Donepezil 3. Bethanechol 4. Tacrine 37. Activation of central alpha2 receptors results in inhibition of cardioacceleration and ______________ centers in the brain. 1. Vasodilation 2. Vasoconstriction 3. Cardiovascular 4. Respiratory Chapter 15. Drugs Affecting the Central Nervous System 1. Sarah, a 42-year-old female, requests a prescription for an anorexiant to treat her obesity. A trial of phentermine is prescribed. Prescribing precautions include: 1. Understanding that obesity is a contraindication to prescribing phentermine 2. Anorexiants may cause tolerance and should only be prescribed for 6 months 3. Patients should be monitored for postural hypotension 4. Renal function should be monitored closely while on anorexiants 2. Before prescribing phentermine to Sarah, a thorough drug history should be taken including assessing for the use of serotonergic agents such as selective serotonin reuptake inhibitors (SSRIs) and St John’s wort due to: 1. Additive respiratory depression risk 2. Additive effects affecting liver function 3. The risk of serotonin syndrome 4. The risk of altered cognitive functioning 3. Antonia is a 3-year-old child who has a history of status epilepticus. Along with her routine antiseizure medication, she should also have a home prescription for_________ to be used for an episode of status epilepticus. 1. IV phenobarbital 2. Rectal diazepam (Diastat) 3. IV phenytoin (Dilantin) 4. Oral carbamazepine (Tegretol) 4. Rabi is being prescribed phenytoin for seizures. Monitoring includes assessing: 1. For phenytoin hypersensitivity syndrome 3 to 8 weeks after starting treatment 2. For pedal edema throughout therapy 3. Heart rate at each visit and consider altering therapy if heart rate is less than 60 bpm 4. For vision changes, such as red-green blindness, at least annually 5. Dwayne has recently started on carbamazepine to treat seizures. He comes to see you and you note that while his carbamazepine levels had been in the therapeutic range, they are now low. The possible cause for the low carbamazepine levels include: 1. Dwayne hasn’t been taking his carbamazepine because it causes insomnia. 2. Carbamazepine auto-induces metabolism, leading to lower levels in spite of good compliance. 3. Dwayne was not originally prescribed the correct amount of carbamazepine. 4. Carbamazepine is probably not the right antiseizure medication for Dwayne. 6. Carbamazepine has a Black Box Warning due to life-threatening: 1. Renal toxicity, leading to renal failure 2. Hepatotoxicity, leading to liver failure 3. Dermatologic reaction, including Steven’s Johnson and toxic epidermal necrolysis 4. Cardiac effects, including supraventricular tachycardia 7. Long-term monitoring of patients who are taking carbamazepine includes: 1. Routine troponin levels to assess for cardiac damage 2. Annual eye examinations to assess for cataract development 3. Monthly pregnancy tests for all women of childbearing age 4. Complete blood count every 3 to 4 months 8. Six-year-old Lucy has recently been started on ethosuximide (Zarontin) for seizures. She should be monitored for: 1. Increased seizure activity, as this drug may auto-induce seizures 2. Altered renal function, including renal failure 3. Blood dyscrasias, which are uncommon but possible 4. Central nervous system excitement, leading to insomnia 9. Sook has been prescribed gabapentin to treat neuropathic pain and is complaining of feeling depressed and having “strange” thoughts. The appropriate initial action would be: 1. Increase her dose 2. Assess for suicidal ideation 3. Discontinue the medication immediately 4. Decrease her dose to half then slowly titrate up the dose 10. Selma, who is overweight, recently started taking topiramate for seizures and at her follow-up visit you note she has lost 3 kg. The appropriate action would be: 1. Tell her to increase her caloric intake to counter the effects of the topiramate. 2. Consult with a neurologist, as this is not a common adverse effect of topiramate. 3. Decrease her dose of topiramate. 4. Reassure her that this is a normal side effect of topiramate and continue to monitor her weight. 11. Monitoring of a patient on gabapentin to treat seizures includes: 1. Routine therapeutic drug levels every 3 to 4 months 2. Assessing for dermatologic reactions, including Steven’s Johnson 3. Routine serum electrolytes, especially in hot weather 4. Recording seizure frequency, duration, and severity 12. Scott’s seizures are well controlled on topiramate and he wants to start playing baseball. Education for Scott regarding his topiramate includes: 1. He should not play sports due to the risk of increased seizures 2. He should monitor his temperature and ability to sweat in the heat while playing 3. Reminding him that he may need higher dosages of topiramate when exercising 4. Encouraging him to use sunscreen due to photosensitivity from topiramate 13. Cara is taking levetiracetam (Keppra) to treat seizures. Routine education for levetiracetam includes reminding her: 1. To not abruptly discontinue levetiracetam due to risk for withdrawal seizures 2. To wear sunscreen due to photosensitivity from levetiracetam 3. To get an annual eye exam while on levetiracetam 4. To report weight loss if it occurs 14. Levetiracetam has known drug interactions with: 1. Combined oral contraceptives 2. Carbamazepine 3. Warfarin 4. Few, if any, drugs 15. Zainab is taking lamotrigine (Lamictal) and presents to the clinic with fever and lymphadenopathy. Initial evaluation and treatment includes: 1. Reassuring her she has a viral infection and to call if she isn’t better in 4 or 5 days 2. Ruling out a hypersensitivity reaction that may lead to multi-organ failure 3. Rapid strep test and symptomatic care if strep test is negative 4. Observation only, with further assessment if she worsens 16. Samantha is taking lamotrigine (Lamictal) for her seizures and requests a prescription for combined oral contraceptives (COCs), which interact with lamotrigine and may cause: 1. Contraceptive failure 2. Excessive weight gain 3. Reduced lamotrigine levels, requiring doubling the dose of lamotrigine 4. Induction of estrogen metabolism, requiring higher estrogen content OCs be prescribed 17. The tricyclic antidepressants should be prescribed cautiously in patients with: 1. Eczema 2. Asthma 3. Diabetes 4. Heart disease 18. A 66-year-old male was prescribed phenelzine (Nardil) while in an acute psychiatric unit for recalcitrant depression. The NP managing his primary health care needs to understand the following regarding phenelzine and other monoamine oxidase inhibitors (MAOIs): 1. He should not be prescribed any serotonergic drug such as sumatriptan (Imitrex) 2. MAOIs interact with many common foods, including yogurt, sour cream, and soy sauce 3. Symptoms of hypertensive crisis (headache, tachycardia, sweating) require immediate treatment 4. All of the above 19. Taylor is a 10-year-old child diagnosed with major depression. The appropriate first-line antidepressant for children is: 1. Fluoxetine 2. Fluvoxamine 3. Sertraline 4. Escitalopram 20. Suzanne is started on paroxetine (Paxil), a selective serotonin reuptake inhibitor (SSRI), for depression. Education regarding her antidepressant includes: 1. SSRIs may take 2 to 6 weeks before she will have maximum drug effects. 2. Red-green color blindness may occur and should be reported. 3. If she experiences dry mouth or heart rates greater than 80, she should stop taking the drug immediately. 4. She should eat lots of food high in fiber to prevent constipation. 21. Cecilia presents with depression associated with complaints of fatigue, sleeping all the time, and lack of motivation. An appropriate initial antidepressant for her would be: 1. Fluoxetine (Prozac) 2. Paroxetine (Paxil) 3. Amitriptyline (Elavil) 4. Duloxetine (Cymbalta) 22. Jake, a 45-year-old patient with schizophrenia, was recently hospitalized for acute psychosis due to medication noncompliance. He was treated with IM long-acting haloperidol. Besides monitoring his schizophrenia symptoms, the patient should be assessed by his primary care provider: 1. For excessive weight loss 2. With the Abnormal Involuntary Movement Scale (AIMS) for extrapyramidal symptoms (EPS) 3. Monthly for tolerance to the haloperidol 4. Only by the mental health provider, as most NPs in primary care do not care for mentally ill patients 23. Anticholinergic agents, such as benztropine (Cogentin), may be given with a phenothiazine to: 1. Reduce the chance of tardive dyskinesia 2. Potentiate the effects of the drug 3. Reduce the tolerance that tends to occur 4. Increase central nervous system (CNS) depression 24. Patients who are prescribed olanzapine (Zyprexa) should be monitored for: 1. Insomnia 2. Weight gain 3. Hypertension 4. Galactorrhea 25. A 19-year-old male was started on risperidone. Monitoring for risperidone includes observing for common side effects, including: 1. Bradykinesia, akathisia, and agitation 2. Excessive weight gain 3. Hypertension 4. Potentially fatal agranulocytosis 26. In choosing a benzodiazepam to treat anxiety the prescriber needs to be aware of the possibility of dependence. The benzodiazepam with the greatest likelihood of rapidly developing dependence is: 1. Chlordiazepoxide (Librium) 2. Clonazepam (Klonopin) 3. Alprazolam (Xanax) 4. Oxazepam (Serax) 27. A patient with anxiety and depression may respond to: 1. Duloxetine (Cymbalta) 2. Fluoxetine (Prozac) 3. Oxazepam (Serax) 4. Buspirone (Buspar) and an SSRI combined 28. When prescribing temazepam (Restoril) for insomnia, patient education includes: 1. Take temazepam nightly approximately 15 minutes before bedtime. 2. Temazepam should not be used more than three times a week for less than 3 months. 3. Drinking 1 ounce of alcohol will cause additive effects and the patient will sleep better. 4. Exercise for at least 30 minutes within 2 hours of bedtime to enhance the effects of temazepam. 29. Patients should be instructed regarding the rapid onset of zolpidem (Ambien) because: 1. Zolpidem should be taken just before going to bed. 2. Zolpidem may cause dry mouth and constipation. 3. Patients may need to double the dose for effectiveness. 4. They should stop drinking alcohol at least 30 minutes before taking zolpidem. 30. One major drug used to treat bipolar disease is lithium. Because lithium has a narrow therapeutic range, it is important to recognize symptoms of toxicity, such as: 1. Orthostatic hypotension 2. Agitation and irritability 3. Drowsiness and nausea 4. Painful urination and abdominal distention 31. Tom is taking lithium for bipolar disorder. He should be taught to: 1. Take his lithium with food 2. Eat a diet with consistent levels of salt (sodium) 3. Drink at least 2 quarts of water if he is in a hot environment 4. Monitor blood glucose levels 32. Cynthia is taking valproate (Depakote) for seizures and would like to get pregnant. What advice would you give her? 1. Valproate is safe during all trimesters of pregnancy. 2. She can get pregnant while taking valproate, but she should take adequate folic acid. 3. Valproate is not safe at any time during pregnancy. 4. Valproate is a known teratogen, but may be taken after the first trimester if necessary. 33. When prescribing an opioid analgesic such as acetaminophen and codeine (Tylenol #3), instructions to the patient should include: 1. The medication may cause sedation and they should not drive. 2. Constipation is a common side effect and they should increase fluids and fiber. 3. Patients should not take any other acetaminophen-containing medications at the same time. 4. All of the above 34. Kirk sprained his ankle and is asking for pain medication for his mild-to-moderate pain. The appropriate first-line medication would be: 1. Ibuprofen (Advil) 2. Acetaminophen with hydrocodone (Vicodin) 3. Oxycodone (Oxycontin) 4. Oral morphine (Roxanol) 35. Kasey fractured his ankle in two places and is asking for medication for his pain. The appropriate first-line medication would be: 1. Ibuprofen (Advil) 2. Acetaminophen with hydrocodone (Vicodin) 3. Oxycodone (Oxycontin) 4. Oral morphine (Roxanol) 36. Jack, age 8, has attention deficit disorder (ADD) and is prescribed methylphenidate (Ritalin). He and his parents should be educated about the side effects of methylphenidate, which are: 1. Slurred speech and insomnia 2. Bradycardia and confusion 3. Dizziness and orthostatic hypotension 4. Insomnia and decreased appetite 37. Monitoring for a child on methylphenidate for attention deficit hyperactivity disorder (ADHD) includes: 1. ADHD symptoms 2. Routine height and weight checks 3. Amount of methylphenidate being used 4. All of the above 38. When prescribing Adderall (amphetamine and dextroamphetamine) to adults with ADHD the nurse practitioner will need to monitor: 1. Blood pressure 2. Blood glucose levels 3. Urine ketone levels 4. Liver function Chapter 16. Drugs Affecting the Cardiovascular and Renal Systems ____ 1. Ray has been diagnosed with hypertension and an angiotensin-converting enzyme inhibitor is determined to be needed. Prior to prescribing this drug, the NP should assess for: 1. Hypokalemia 2. Impotence 3. Decreased renal function 4. Inability to concentrate ____ 2. Angiotensin-converting enzyme inhibitors are the drug of choice in treating hypertension in diabetic patients because they: 1. Improve insulin sensitivity 2. Improve renal hemodynamics 3. Reduce the production of angiotensin II 4. All of the above ____ 3. A potentially life-threatening adverse response to angiotensin-converting enzyme inhibitors is angioedema. Which of the following statements is true about this adverse response? 1. Swelling of the tongue or hoarseness are the most common symptoms. 2. It appears to be related to the decrease in aldosterone production. 3. Presence of a dry, hacky cough indicates a high risk for this adverse response. 4. Because it takes time to build up a blood level, it occurs after being on the drug for about 1 week. ____ 4. Angiotensin-converting enzyme inhibitors are useful in a variety of disorders. Which of the following statements are true about both its usefulness in the disorder and the reason for its use? 1. Stable angina because it decreases the thickening of vascular walls due to decreased modified release. 2. Heart failure because it reduces remodeling of injured myocardial tissues. 3. Both 1 and 2 are true and the reasons are correct. 4. Both 1 and 2 are true but the reasons are wrong. 5. Neither 1 nor 2 are true. ____ 5. Despite good blood pressure control, an NP might change a patient’s drug from an angiotensin-converting enzyme (ACE) inhibitor to an angiotensin II receptor blocker (ARB) because the ARB: 1. Is stronger than the ACE inhibitor 2. Does not produce a dry, hacky cough 3. Has no effect on the renal system 4. Reduces sodium and water retention ____ 6. While taking an angiotensin II receptor blocker (ARB), patients need to avoid certain over-the-counter drugs without first consulting the provider because: 1. Cimetidine is metabolized by the CYP 3A4 isoenzymes 2. Nonsteroidal anti-inflammatory drugs reduce prostaglandin levels 3. Both 1 and 2 4. Neither 1 nor 2 ____ 7. Laboratory monitoring for patients on angiotensin-converting enzyme inhibitors or angiotensin II receptor blockers should include: 1. White blood cell counts with the drug dosage increased for elevations above 10,000 feet 2. Liver function tests with the drug dosage stopped for alanine aminotransferase values twice that of normal 3. Serum creatinine levels with the drug dosage reduced for values greater than 2.5 mg/dL 4. Serum glucose levels with the drug dosage increased for levels greater than 120 mg/dL ____ 8. Jacob has hypertension, for which a calcium channel blocker has been prescribed. This drug helps control blood pressure because it: 1. Decreases the amount of calcium inside the cell 2. Reduces stroke volume 3. Increases the activity of the Na+/K+/ATPase pump indirectly 4. Decreases heart rate ____ 9. Which of the following adverse effects may occur due to a dihydropyridine-type calcium channel blocker? 1. Bradycardia 2. Hepatic impairment 3. Increased contractility 4. Edema of the hands and feet ____ 10. Patient teaching related to amlodipine includes: 1. Increase calcium intake to prevent osteoporosis from a calcium blockade. 2. Do not crush the tablet; it must be given in liquid form if the patient has trouble swallowing it. 3. Avoid grapefruit juice as it affects the metabolism of this drug. 4. Rise slowly from a supine position to reduce orthostatic hypotension. ____ 11. Vera, age 70, has isolated systolic hypertension. Calcium channel blocker dosages for her should be: 1. Started at about half the usual dosage 2. Not increased over the usual dosage for an adult 3. Given once daily because of memory issues in the older adult 4. Withheld if she experiences gastroesophageal reflux ____ 12. Larry has heart failure, which is being treated with digoxin because it exhibits: 1. Negative inotropism 2. Positive chronotropism 3. Both 1 and 2 4. Neither 1 nor 2 ____ 13. Furosemide is added to a treatment regimen for heart failure that includes digoxin. Monitoring for this combination includes: 1. Hemoglobin 2. Serum potassium 3. Blood urea nitrogen 4. Serum glucose ____ 14. Which of the following create a higher risk for digoxin toxicity? Both the cause and the reason for it must be correct. 1. Older adults because of reduced renal function 2. Administration of aldosterone antagonist diuretics because of decreased potassium levels 3. Taking an antacid for gastroesophageal reflux disease because it increases the absorption of digoxin 4. Doses between 0.25 and 0.5 mg/day ____ 15. Serum digoxin levels are monitored for potential toxicity. Monitoring should occur: 1. Within 6 hours of the last dose 2. Because a reference point is needed in adjusting a dose 3. After three half-lives from the starting of the drug 4. When a patient has stable renal function ____ 16. Rodrigo has been prescribed procainamide after a myocardial infarction. He is monitored for dyspnea, jugular venous distention, and peripheral edema because they may indicate: 1. Widening of the area of infarction 2. Onset of congestive heart failure 3. An electrolyte imbalance involving potassium 4. Renal dysfunction ____ 17. Which of the following is true about procainamide and its dosing schedule? 1. It produces bradycardia and should be used cautiously in patients with cardiac conditions that a slower heart rate might worsen. 2. Gastrointestinal adverse effects are common so the drug should be taken with food. 3. Adherence can be improved by using a sustained release formulation that can be given once daily. 4. Doses of this drug should be taken evenly spaced around the clock to keep an even blood level. ____ 18. Amiodarone has been prescribed in a patient with a supraventricular dysrhythmia. Patient teaching should include all of the following EXCEPT: 1. Notify your health-care provider immediately if you have visual change. 2. Monitor your own blood pressure and pulse daily. 3. Take a hot shower or bath if you feel dizzy. 4. Use a sunscreen on exposed body surfaces. ____ 19. The NP orders a thyroid panel for a patient on amiodarone. The patient tells the NP that he does not have thyroid disease and wants to know why the test is ordered. Which is a correct response? 1. Amiodarone inhibits an enzyme that is important in making thyroid hormone and can cause hypothyroidism. 2. Amiodarone damages the thyroid gland and can result in inflammation of that gland, causing hyperthyroidism. 3. Amiodarone is a broad spectrum drug with many adverse effects. Many different tests need to be done before it is given. 4. Amiodarone can cause corneal deposits in up to 25% of patients. ____ 20. Isosorbide dinitrate is prescribed for a patient with chronic stable angina. This drug is administered twice daily, but the schedule is 7 a.m. and 2 p.m. because: 1. It is a long-acting drug with potential for toxicity. 2. Nitrate tolerance can develop. 3. Orthostatic hypotension is a common adverse effect. 4. It must be taken with milk or food. ____ 21. Art is a 55-year-old smoker who has been diagnosed with angina and placed on nitrates. He complains of headaches after using his nitrate. An appropriate reply might be: 1. This is a parasympathetic response to the vasodilating effects of the drug. 2. Headaches are common side effects with these drugs. How severe are they? 3. This is associated with your smoking. Let’s work on having you stop smoking. 4. This is not related to your medication. Are you under a lot of stress? ____ 22. In teaching about the use of sublingual nitroglycerine, the patient should be instructed: 1. To swallow the tablet with a full glass of water 2. To place one tablet under the tongue if chest pain occurs and allow it to dissolve 3. To take one tablet every 5 minutes until the chest pain goes away 4. That it should “burn” when placed under the tongue or it is no longer effective ____ 23. Donald has been diagnosed with hyperlipidemia. Based on his lipid profile, atorvastatin is prescribed. Rhabdomyolysis is a rare but serious adverse response to this drug. Donald should be told to: 1. Become a vegetarian because this disorder is associated with eating red meat. 2. Stop taking the drug if abdominal cramps and diarrhea develop. 3. Report muscle weakness or tenderness and dark urine to his provider immediately. 4. Expect “hot flash” sensations during the first 2 weeks of therapy. ____ 24. Which of the following diagnostic studies would NOT indicate a problem related to a reductase inhibitor? 1. Elevated serum transaminase 2. Increased serum creatinine 3. Elevated creatinine kinase 4. Increased white blood cell counts ____ 25. Because of the pattern of cholesterol synthesis, reductase inhibitors are given: 1. In the evening in a single daily dose 2. Twice daily in the morning and the evening 3. With each meal and at bedtime 4. In the morning before eating ____ 26. Janice has elevated LDL, VLDL, and triglyceride levels. Niaspan, an extended-release form of niacin, is chosen to treat her hyperlipidemia. Due to its metabolism and excretion, which of the following laboratory results should be monitored? 1. Serum alanine aminotransferase 2. Serum amylase 3. Serum creatinine 4. Phenylketonuria ____ 27. Niaspan is less likely to cause which side effect that is common to niacin? 1. Gastrointestinal irritation 2. Cutaneous flushing 3. Dehydration 4. Headaches ____ 28. Dulcea has type 2 diabetes and a high triglyceride level. She has gemfibrozil prescribed to treat her hypertriglyceridemia. A history of which of the following might contraindicate the use of this drug? 1. Reactive airway disease/asthma 2. Inflammatory bowel disease 3. Allergy to aspirin 4. Gallbladder disease ____ 29. Many patients with hyperlipidemia are treated with more than one drug. Combining a fibric acid derivative such as gemfibrozil with which of the following is not recommended? The drug and the reason must both be correct for the answer to be correct. 1. Reductase inhibitors, due to an increased risk for rhabdomyolysis 2. Bile-acid sequestering resins, due to interference with folic acid absorption 3. Grapefruit juice, due to interference with metabolism 4. Niacin, due to decreased gemfibrozil activity ____ 30. Felicity has been prescribed colestipol to treat her hyperlipidemia. Unlike other anti-lipidemics, this drug: 1. Blocks synthesis of cholesterol in the liver 2. Exchanges chloride ions for negatively charged acids in the bowel 3. Increases HDL levels the most among the classes 4. Blocks the lipoprotein lipase pathway ____ 31. Because of their site of action, bile acid sequestering resins: 1. Should be administered separately from other drugs by at least 4 hours 2. May increase the risk for bleeding 3. Both 1 and 2 4. Neither 1 nor 2 ____ 32. Colestipol comes in a powdered form. The patient is taught to: 1. Take the powder dry and follow it with at least 8 ounces of water 2. Take it with a meal to enhance its action on fatty food 3. Mix the powder with 4 to 6 ounces of milk or fruit juice 4. Take after the evening meal to coincide with cholesterol synthesis ____ 33. The choice of diuretic to use in treating hypertension is based on: 1. Presence of diabetes with loop diuretics being used for these patients 2. Level of kidney function with a thiazide diuretic being used for an estimated glomerular filtration rate higher than the mid-40mL/min range 3. Ethnicity with aldosterone antagonists best for African Americans and older adults 4. Presence of hyperlipidemia with higher doses needed for patients with LDL above 130 mg/dL ____ 34. Direct renin inhibitors have the following properties. They: 1. Are primarily generic drugs 2. Are a renin-angiotensin-aldosterone system (RAAS) medication that is safe during pregnancy 3. Can be used with an angiotensin-converting enzyme and angiotensin II receptor blocker medications for stronger impact 4. “Shut down” the entire RAAS cycle ____ 35. When comparing angiotensin-converting enzyme (ACE) and angiotensin II receptor blocker (ARB) medications, which of the following holds true? 1. Both have major issues with a dry, irritating cough 2. Both contribute to some retention of potassium 3. ARBs have a stronger impact on hypertension control than ACE medications 4. ARBs have stronger diabetes mellitus renal protection properties than ACE medications ____ 36. What does the provider understand about the issue of “Diabetic Renal Protection” with angiotensin-converting enzyme (ACE) medications? Diabetes mellitus patients: 1. Have a reduced rate of renal progression, but still need to be discontinued when advanced renal issues present 2. Who start these medications never progress to renal nephropathy 3. With early renal dysfunction will see it reverse when on ACE medications 4. Without renal issues are the only ones who benefit from ACE protection ____ 37. What dermatological issue is linked to Amiodarone use? 1. Increased risk of basal cell carcinoma 2. Flare up of any prior psoriasis problems 3. Development of plantar warts 4. Progressive change of skin tone toward a blue spectrum ____ 38. Commercials on TV for erectile dysfunction (ED) medications warn about mixing them with nitrates. Why? 1. Increased risk of priapism 2. Profound hypotension 3. Development of blue discoloration to the visual field 4. Inactivation of the ED medication effect Chapter 17. Drugs Affecting the Respiratory System ____ 1. Digoxin levels need to be monitored closely when the following medication is started: 1. Loratadine 2. Diphenhydramine 3. Ipratropium 4. Albuterol ____ 2. Patients with pheochromocytoma should avoid which of the following classes of drugs because of the possibility of developing hypertensive crisis? 1. Expectorants 2. Beta-2-agonists 3. Antitussives 4. Antihistamines ____ 3. Harold, a 42-year-old African American, has moderate persistent asthma. Which of the following asthma medications should be used cautiously, if at all? 1. Betamethasone, an inhaled corticosteroid 2. Salmeterol, an inhaled long-acting beta-agonist 3. Albuterol, a short-acting beta-agonist 4. Montelukast, a leukotriene modifier ____ 4. Long-acting beta-agonists (LTBAs) received a Black Box Warning from the U.S. Food and Drug Administration due to the: 1. Risk of life-threatening dermatological reactions 2. Increased incidence of cardiac events when LTBAs are used 3. Increased risk of asthma-related deaths when LTBAs are used 4. Risk for life-threatening alterations in electrolytes ____ 5. The bronchodilator of choice for patients taking propranolol is: 1. Albuterol 2. Pirbuterol 3. Formoterol 4. Ipratropium ____ 6. James is a 52-year-old overweight smoker taking theophylline for his persistent asthma. He tells his provider he is going to start the Atkin’s diet for weight loss. The appropriate response would be: 1. Congratulate him on making a positive change in his life. 2. Recommend he try stopping smoking instead of the Atkin’s diet. 3. Schedule him for regular testing of serum theophylline levels during his diet due to increased excretion of theophylline. 4. Decrease his theophylline dose because a high-protein diet may lead to elevated theophylline levels. ____ 7. Li takes theophylline for his persistent asthma and calls the office with a complaint of nausea, vomiting, and headache. The best advice for him would be to: 1. Reassure him this is probably a viral infection and should be better soon 2. Have him seen the same day for an assessment and theophylline level 3. Schedule him for an appointment in 2 to 3 days, which he can cancel if he is better 4. Order a theophylline level at the laboratory for him ____ 8. Tiotropium bromide (Spiriva) is an inhaled anticholinergic: 1. Used for the treatment of chronic obstructive pulmonary disease (COPD) 2. Used in the treatment of asthma 3. Combined with albuterol for treatment of asthma exacerbations 4. Combined with fluticasone for the treatment of persistent asthma ____ 9. Christy has exercise-induced and mild persistent asthma and is prescribed two puffs of albuterol 15 minutes before exercise and as needed for wheezing. One puff per day of beclomethasone (QVAR) is also prescribed. Teaching regarding her inhalers includes: 1. Use one to two puffs of albuterol per day to prevent an attack with no more than eight puffs per day 2. Beclomethasone needs to be used every day to treat her asthma 3. Report any systemic side effects she is experiencing, such as weight gain 4. Use the albuterol metered-dose inhaler (MDI) immediately after her corticosteroid MDI to facilitate bronchodilation ____ 10. When prescribing montelukast (Singulair) for asthma, patients or parents of patients should be instructed: 1. Montelukast twice a day is started when there is an asthma exacerbation. 2. Patients may experience weight gain on montelukast. 3. Aggression, anxiety, depression, and/or suicidal thoughts may occur when taking montelukast. 4. Lethargy and hypersomnia may occur when taking montelukast. ____ 11. Montelukast (Singulair) may be prescribed for: 1. A 6-year-old child with exercise-induced asthma 2. A 2-year-old child with moderate persistent asthma 3. An 18-month-old child with seasonal allergic rhinitis 4. None of the above; montelukast is not approved for use in children ____ 12. The known drug interactions with the inhaled corticosteroid beclomethasone (QVAR) include: 1. Albuterol 2. MMR vaccine 3. Insulin 4. None of the above ____ 13. When educating patients who are starting on inhaled corticosteroids, the provider should tell them that: 1. They need to get any live vaccines before starting the medication. 2. Inhaled corticosteroids need to be used daily during asthma exacerbations to be effective. 3. Patients should rinse their mouths out after using the inhaled corticosteroid to prevent thrush. 4. They can triple the dose number of inhalations of medication during colds to prevent needing systemic steroids. ____ 14. Patients with allergic rhinitis may benefit from a prescription of: 1. Fluticasone (Flonase) 2. Cetirizine (Zyrtec) 3. OTC cromolyn nasal spray (Nasalcrom) 4. Any of the above ____ 15. Howard is a 72-year-old male who occasionally takes diphenhydramine for his seasonal allergies. Monitoring for this patient taking diphenhydramine would include assessing for: 1. Urinary retention 2. Cardiac output 3. Peripheral edema 4. Skin rash ____ 16. First-generation antihistamines such as loratadine (Claritin) are prescribed for seasonal allergies because they are: 1. More effective than first-generation antihistamines 2. Less sedating than the first-generation antihistamines 3. Prescription products, therefore are covered by insurance 4. Able to be taken with central nervous system (CNS) sedatives, such as alcohol ____ 17. When recommending dimenhydrinate (Dramamine) to treat motion sickness, patients should be instructed to: 1. Take the dimenhydrinate after they get nauseated 2. Drink lots of water while taking the dimenhydrinate 3. Take the dimenhydrinate 15 minutes before it is needed 4. Double the dose if one tablet is not effective ____ 18. Decongestants such as pseudoephedrine (Sudafed): 1. Are Schedule III drugs in all states 2. Should not be prescribed or recommended for children under 4 years of age 3. Are effective in treating the congestion children experience with the common cold 4. May cause drowsiness in patients of all ages ____ 19. Cough and cold medications that contain a sympathomimetic decongestant such as phenylephrine should be used cautiously in what population: 1. Older adults 2. Hypertensive patients 3. Infants 4. All of the above ____ 20. Martin is a 60-year-old patient with hypertension. The first-line decongestant to prescribe would be: 1. Oral pseudoephedrine 2. Oral phenylephrine 3. Nasal oxymetazoline 4. Nasal azelastine ____ 21. The first-line treatment for cough related to an upper respiratory tract infection (URI) in a 5-year-old child is: 1. Fluids and symptomatic care 2. Dextromethorphan and guaifenesin syrup (Robitussin DM for Kids) 3. Guaifenesin and codeine syrup (Tussin AC) Chapter 18. Drugs Affecting the Hematopoietic System ____ 1. Kenneth is taking warfarin and is asking about what he can take for minor aches and pains. The best recommendation is: 1. Ibuprofen 400 mg three times a day 2. Acetaminophen, not to exceed 4 grams per day 3. Prescribe acetaminophen with codeine 4. Aspirin 640 mg three times a day ____ 2. Juanita had a deep vein thrombosis (DVT) and was on heparin in the hospital and was discharged on warfarin. She asks her primary care provider NP why she was getting both medications while in the hospital. The best response is to: 1. Contact the hospitalist as this is not the normal guideline for prescribing these two medications and she may have had a more complicated case. 2. Explain that warfarin is often started while a patient is still on heparin because warfarin takes a few days to reach effectiveness. 3. Encourage the patient to contact the Customer Service department at the hospital as this was most likely a medication error during her admission. 4. Draw anticoagulation studies to make sure she does not have dangerously high bleeding times. ____ 3. The safest drug to use to treat pregnant women who require anticoagulant therapy is: 1. Low-molecular-weight heparin 2. Warfarin 3. Aspirin 4. Heparin ____ 4. The average starting dose of warfarin is 5 mg daily. Higher doses of 7.5 mg daily should be considered in which patients? 1. Pregnant women 2. Elderly men 3. Overweight or obese patients 4. Patients with multiple comorbidities ____ 5. Cecil and his wife are traveling to Southeast Asia on vacation and he has come into the clinic to review his medications. He is healthy with only mild hypertension that is well controlled. He asks about getting “a shot” to prevent blood clots like his friend Ralph did before international travel. The correct respond would be: 1. Administer one dose of low-molecular weight heparin 24 hours before travel. 2. Prescribe one dose of warfarin to be taken the day of travel. 3. Consult with a hematologist regarding a treatment plan for Cecil. 4. Explain that Cecil is not at high risk of a blood clot and provide education about how to prevent blood clots while traveling. ____ 6. Robert, age 51 years, has been told by his primary care provider (PCP) to take an aspirin a day. Why would this be recommended? 1. He has arthritis and this will help with the inflammation and pain. 2. Aspirin has anti-platelet activity and prevents clots that cause heart attacks. 3. Aspirin acidifies the urine and he needs this for prostrate health. 4. He has a history of GI bleed, and one aspirin a day is a safe dosage. ____ 7. Sally has been prescribed aspirin 320 mg per day for her atrial fibrillation. She also takes aspirin four or more times a day for arthritis pain. What are the symptoms of aspirin toxicity for which she would need to be evaluated? 1. Tinnitus 2. Diarrhea 3. Hearing loss 4. Photosensitivity ____ 8. Patient education when prescribing clopidogrel includes: 1. Do not take any herbal products without discussing it with the provider. 2. Monitor urine output closely and contact the provider if it decreases. 3. Clopidogrel can be constipating, use a stool softener if needed. 4. The patient will need regular anticoagulant studies while on clopidogrel. ____ 9. For patients taking warfarin, INRs are best drawn: 1. Monthly throughout therapy 2. Three times a week throughout therapy 3. Two hours after the last dose of warfarin to get an accurate peak level 4. In the morning if the patient takes their warfarin at night ____ 10. Patients receiving heparin therapy require monitoring of: 1. Platelets every 2 to 3 days for thrombocytopenia that may occur on day 4 of therapy 2. Electrolytes for elevated potassium levels in the first 24 hours of therapy 3. INR throughout therapy to stay within the range of 2.0 4. Blood pressure for hypertension that may occur in the first 2 days of treatment ____ 11. The routine monitoring recommended for low molecular weight heparin is: 1. INR every 2 days until stable, then weekly 2. aPTT every week while on therapy 3. Factor Xa levels if the patient is pregnant 4. White blood cell count every 2 weeks ____ 12. When writing a prescription for warfarin it is common to write _________ on the prescription. 1. OK to substitute for generic 2. The brand name of warfarin and Do Not Substitute 3. PRN refills 4. Refills for 1 year ____ 13. Education of patients who are taking warfarin includes discussing their diet. Instructions include: 1. Avoiding all vitamin K-containing foods 2. Avoiding high-vitamin K-containing foods 3. Increasing intake of iron-containing foods 4. Making sure they eat 35 grams of fiber daily ____ 14. Patients who are being treated with epoetin alfa need to be monitored for the development of: 1. Thrombocytopenia 2. Neutropenia 3. Hypertension 4. Gout ____ 15. The FDA issued a safety announcement regarding the use of erythropoiesis-stimulating agents (ESAs) in 2010 with the recommendation that: 1. ESAs no longer be prescribed to patients with chronic renal failure 2. The risk of tumor development be explained to cancer patients on ESA therapy 3. Patients should no longer receive ESA therapy to prepare for allogenic transfusions 4. ESAs be prescribed only to patients younger than age 60 years ____ 16. When patients are started on darbepoetin alfa (Aranesp) they need monitoring of their blood counts to determine a dosage adjustment in: 1. 6 weeks if they are a cancer patient 2. 1 week if they have chronic renal failure 3. 2 weeks if they are taking it for allogenic transfusion 4. Each week throughout therapy ____ 17. Jim is having a hip replacement surgery and would like to self-donate blood for the surgery. In addition to being prescribed epogen alpha he should also be prescribed: 1. Folic acid to prevent megaloblastic anemia 2. Iron, to start when the epogen starts 3. An antihypertensive to counter the adverse effects of epogen 4. Vitamin B12 to prevent pernicious anemia ____ 18. Monitoring for a patient being prescribed iron for iron deficiency anemia includes: 1. Reticulocyte count 1 week after therapy is started 2. Complete blood count every 2 weeks throughout therapy 3. Hemoglobin level at 1 week of therapy 4. INR weekly throughout therapy ____ 19. Patient education regarding taking iron replacements includes: 1. Doubling the dose if they miss a dose to maintain therapeutic levels 2. Taking the iron with milk or crackers if it upsets their stomach 3. Iron is best taken on an empty stomach with juice 4. Antacids such as Tums may help the upset stomach caused by iron therapy ____ 20. Patients with pernicious anemia require treatment with: 1. Iron 2. Folic acid 3. Epogen alpha 4. Vitamin B12 ____ 21. The first laboratory value indication that vitamin B12 therapy is adequately treating pernicious anemia is: 1. Hematocrit levels start to rise 2. Hemoglobin levels return to normal 3. Reticulocyte count begins to rise 4. Vitamin B12 levels return to normal ____ 22. Patients who are beginning therapy with vitamin B12 need to be monitored for: 1. Hypertensive crisis that may occur in the first 36 hours 2. Hypokalemia that occurs in the first 48 hours 3. Leukopenia that occurs at 1 to 3 weeks of therapy 4. Thrombocytopenia that may occur at any time in therapy Chapter 20. Drugs Affecting the Gastrointestinal System ____ 1. Many patients self-medicate with antacids. Which patients should be counseled to not take calcium carbonate antacids without discussing it with their provider or a pharmacist first? 1. Patients with kidney stones 2. Pregnant patients 3. Patients with heartburn 4. Postmenopausal women ____ 2. Patients taking antacids should be educated regarding these drugs, including letting them know that: 1. They may cause constipation or diarrhea 2. Many are high in sodium 3. They should separate antacids from other medications by 1 hour 4. All of the above ____ 3. Kelly has diarrhea and is wondering if she can take loperamide (Imodium) for the diarrhea. Loperamide: 1. Can be given to patients of all ages, including infants and children, for viral gastroenteritis 2. Slows gastric motility and reduces fluid and electrolyte loss from diarrhea 3. Is the treatment of choice for the diarrhea associated with E. coli 0157 4. May be used in pregnancy and by lactating women ____ 4. Bismuth subsalicylate (Pepto Bismol) is a common OTC remedy for gastrointestinal complaints. Bismuth subsalicylate: 1. May lead to toxicity if taken with aspirin 2. Is contraindicated in children with flu-like illness 3. Has antimicrobial effects against bacterial and viral enteropathogens 4. All of the above ____ 5. Hannah will be traveling to Mexico with her church group over spring break to build houses. She is concerned she may develop traveler’s diarrhea. Advice includes following normal food and water precautions as well as taking: 1. Loperamide four times a day throughout the trip 2. Bismuth subsalicylate with each meal and at bedtime 3. A prescription for diphenoxylate with atropine to use if she gets diarrhea 4. None of the above ____ 6. Josie is a 5-year-old patient who presents to the clinic with a 48-hour history of nausea, vomiting, and some diarrhea. She is unable to keep fluids down and her weight is 4 pounds less than her last recorded weight. Besides IV fluids, her exam warrants the use of an antinausea medication. Which of the following would be the appropriate drug to order for Josie? 1. Prochlorperazine (Compazine) 2. Meclizine (Antivert) 3. Promethazine (Phenergan) 4. Ondansetron (Zofran) ____ 7. Jim presents with complaints of “heartburn” that is minimally relieved with Tums (calcium carbonate) and is diagnosed with gastroesophageal reflux disease (GERD). An appropriate first-step therapy would be: 1. Omeprazole (Prilosec) twice a day 2. Ranitidine (Zantac) twice a day 3. Famotidine (Pepcid) once a day 4. Metoclopramide (Reglan) four times a day ____ 8. Patients who are on chronic long-term proton pump inhibitor therapy require monitoring for: 1. Iron deficiency anemia, vitamin B12 and calcium deficiency 2. Folate and magnesium deficiency 3. Elevated uric acid levels leading to gout 4. Hypokalemia and hypocalcemia ____ 9. Sadie is a 72-year-old patient who takes omeprazole for her chronic GERD. Chronic long-term omeprazole use places her at increased risk for: 1. Megaloblastic anemia 2. Osteoporosis 3. Hypertension 4. Strokes ____ 10. Patrick is a 10-year-old patient who presents with uncomfortable constipation. Along with diet changes, a laxative is ordered to provide more rapid relief of constipation. An appropriate choice of medication for a 10-year-old child would be: 1. PEG 3350 (Miralax) 2. Bisacodyl (Dulcolax) suppository 3. Docusate (Colace) suppository 4. Polyethylene glycol electrolyte solution ____ 11. Methylnaltrexone is used to treat constipation in: 1. Patients with functional constipation 2. Patients with irritable bowel syndrome-associated constipation 3. Children with encopresis 4. Opioid-associated constipation ____ 12. An elderly person has been prescribed lactulose for treatment of chronic constipation. Monitoring with long-term treatment would include: 1. Electrolytes, including potassium and chloride 2. Bone mineral density for osteoporosis 3. Magnesium level 4. Liver function Chapter 21. Drugs Affecting the Endocrine System ____ 1. Both men and women experience bone loss with aging. The bones most likely to demonstrate significant loss are: 1. Cortical bones 2. Femoral neck bones 3. Cervical vertebrae 4. Pelvic bones ____ 2. Bisphosphonates treat or prevent osteoporosis by: 1. Inhibiting osteoclastic activity 2. Fostering bone resorption 3. Enhancing calcium uptake in the bone 4. Strengthening the osteoclastic proton pump ____ 3. Prophylactic use of bisphosphonates is recommended for patients with early osteopenia related to long-term use of which of the following drugs? 1. Selective estrogen receptor modulators 2. Aspirin 3. Glucocorticoids 4. Calcium supplements ____ 4. Patients with cystic fibrosis are often prescribed enzyme replacement for pancreatic secretions. Each replacement drug has lipase, protease, and amylase components, but the drug is prescribed in units of: 1. Lipase 2. Protease 3. Amylase 4. Pancreatin ____ 5. Brands of pancreatic enzyme replacement drugs are: 1. Bioequivalent 2. About the same in cost per unit of lipase across brands 3. Able to be interchanged between generic and brand-name products to reduce cost 4. None of the above ____ 6. When given subcutaneously, how long until neutral protamine Hagedorn insulin begins to take effect (onset of action) after administration? 1. 15 to 30 minutes 2. 60 to 90 minutes 3. 3 to 4 hours 4. 6 to 8 hours ____ 7. Hypoglycemia can result from the action of either insulin or an oral hypoglycemic. Signs and symptoms of hypoglycemia include: 1. “Fruity” breath odor and rapid respiration 2. Diarrhea, abdominal pain, weight loss, and hypertension 3. Dizziness, confusion, diaphoresis, and tachycardia 4. Easy bruising, palpitations, cardiac dysrhythmias, and coma ____ 8. Nonselective beta blockers and alcohol create serious drug interactions with insulin because they: 1. Increase blood glucose levels 2. Produce unexplained diaphoresis 3. Interfere with the ability of the body to metabolize glucose 4. Mask the signs and symptoms of altered glucose levels ____ 9. Lispro is an insulin analogue produced by recombinant DNA technology. Which of the following statements about this form of insulin is NOT true? 1. Optimal time of preprandial injection is 15 minutes. 2. Duration of action is increased when the dose is increased. 3. It is compatible with neutral protamine Hagedorn insulin. 4. It has no pronounced peak. ____ 10. The decision may be made to switch from twice daily neutral protamine Hagedorn (NPH) insulin to insulin glargine to improve glycemia control throughout the day. If this is done: 1. The initial dose of glargine is reduced by 20% to avoid hypoglycemia. 2. The initial dose of glargine is 2 to 10 units per day. 3. Patients who have been on high doses of NPH will need tests for insulin antibodies. 4. Obese patients may require more than 100 units per day. ____ 11. When blood glucose levels are difficult to control in type 2 diabetes some form of insulin may be added to the treatment regimen to control blood glucose and limit complication risks. Which of the following statements is accurate based on research? 1. Premixed insulin analogues are better at lowering HbA1C and have less risk for hypoglycemia. 2. Premixed insulin analogues and the newer premixed insulins are associated with more weight gain than the oral antidiabetic agents. 3. Newer premixed insulins are better at lowering HbA1C and postprandial glucose levels than long-acting insulins. 4. Patients who are not controlled on oral agents and have postprandial hyperglycemia can have neutral protamine Hagedorn insulin added at bedtime. ____ 12. Metformin is a primary choice of drug to treat hyperglycemia in type 2 diabetes because it: 1. Substitutes for insulin usually secreted by the pancreas 2. Decreases glycogenolysis by the liver 3. Increases the release of insulin from beta cells 4. Decreases peripheral glucose utilization ____ 13. Prior to prescribing metformin, the provider should: 1. Draw a serum creatinine to assess renal function 2. Try the patient on insulin 3. Tell the patient to increase iodine intake 4. Have the patient stop taking any sulfonylurea to avoid dangerous drug interactions ____ 14. The action of “gliptins” is different from other antidiabetic agents because they: 1. Have a low risk for hypoglycemia 2. Are not associated with weight gain 3. Close ATP-dependent potassium channels in the beta cell 4. Act on the incretin system to indirectly increase insulin production ____ 15. Sitagliptin has been approved for: 1. Monotherapy in once-daily doses 2. Combination therapy with metformin 3. Both 1 and 2 4. Neither 1 nor 2 ____ 16. GLP-1 agonists: 1. Directly bind to a receptor in the pancreatic beta cell 2. Have been approved for monotherapy 3. Speed gastric emptying to decrease appetite 4. Can be given orally once daily ____ 17. Avoid concurrent administration of exenatide with which of the following drugs? 1. Digoxin 2. Warfarin 3. Lovastatin 4. All of the above ____ 18. Administration of exenatide is by subcutaneous injection: 1. 30 minutes prior to the morning meal 2. 60 minutes prior to the morning and evening meal 3. 15 minutes after the evening meal 4. 60 minutes before each meal daily ____ 19. Potentially fatal granulocytopenia has been associated with treatment of hyperthyroidism with propylthiouracil. Patients should be taught to report: 1. Tinnitus and decreased salivation 2. Fever and sore throat 3. Hypocalcemia and osteoporosis 4. Laryngeal edema and difficulty swallowing ____ 20. Elderly patients who are started on levothyroxine for thyroid replacement should be monitored for: 1. Excessive sedation 2. Tachycardia and angina 3. Weight gain 4. Cold intolerance ____ 21. Which of the following is not an indication that growth hormone supplements should be discontinued? 1. Imaging indication of epiphyseal closure 2. Growth curve increases have plateaued 3. Complaints of mild bone pain 4. Achievement of anticipated height goals ____ 22. Besides osteoporosis, IV bisphosphonates are also indicated for: 1. Paget’s Disease 2. Early osteopenia 3. Renal cancer 4. Early closure of cranial sutures ____ 23. What is the role of calcium supplements when patients take bisphosphonates? 1. They must be restricted to allow the medication to work. 2. They must be taken in sufficient amounts to provide foundational elements for bone growth. 3. They must be taken at the same time as the bisphosphonates. 4. They only work with bisphosphonates if daily intake is restricted. ____ 24. Which of the following statements about pancreatic enzymes is true? 1. Dosing may be titrated according to the decrease of steatorrhea. 2. The amount of carbohydrates in the meal drives the amount of enzyme used. 3. The amount of medication used is increased with a cystic fibrosis pulmonary flare. 4. The FDA and Internet-available formulations are bioequivalent. ____ 25. Besides cystic fibrosis, which other medical state may trigger the need for pancreatic enzymes? 1. Paget’s disease 2. Pulmonary cancers 3. Gallbladder surgery 4. Some bariatric surgeries Chapter 22. Drugs Affecting the Reproductive System Multiple Choice Identify the choice that best completes the statement or answers the question. ____ 1. Men who use transdermal testosterone gel (AndroGel) should be advised to avoid: 1. Washing their hands after applying the gel 2. Wearing occlusive clothing while using the gel 3. Exposure to estrogens while using the gel 4. Skin-to-skin contact with pregnant women while using the gel ____ 2. Education when prescribing androgens to male patients includes advising that: 1. Short-term use places the patient at risk for hepatocellular carcinoma. 2. Cholestatic hepatitis and jaundice may occur with low doses of androgens. 3. Gynecomastia is a rare occurrence with the use of androgens. 4. Low sperm levels only occur with long-term use of androgens. ____ 3. Patients who are prescribed exogenous androgens need to be warned that decreased libido: 1. Is an unusual side effect of androgens and should be reported to the provider 2. Is treated with increased doses of androgens, so the patient should let the provider know if he is having problems 3. May be a sign of early prostate cancer and he should make an appointment for a prostate screening exam 4. May occur with androgen therapy ____ 4. The U.S. Food and Drug Administration warns that androgens may cause: 1. Peliosis hepatis 2. Orthostatic hypotension 3. Menstrual irregularities 4. Acne ____ 5. Monitoring for a patient who is using androgens includes evaluation of: 1. Complete blood count and C-reactive protein levels 2. Lipid levels and liver function tests 3. Serum potassium and magnesium levels 4. Urine protein and potassium levels ____ 6. Male patients require ______________________ before and during androgen therapy. 1. A digital prostate exam 2. A Doppler exam of testicular blood flow 3. Urine analysis for proteinuria 4. Serial orthostatic blood pressures ____ 7. Absolute contraindications to estrogen therapy include: 1. History of any type of cancer 2. Clotting disorders 3. History of tension headaches 4. Orthostatic hypotension ____ 8. Postmenopausal women with an intact uterus should not be prescribed: 1. Estrogen/progesterone combination 2. IM medroxyprogesterone (Depo Provera) 3. Estrogen alone 4. Androgens ____ 9. Women who have migraines with an aura should not be prescribed estrogen because of: 1. The interaction between triptans and estrogen, limiting migraine therapy choices 2. An increased incidence of migraines with the use of estrogen 3. An increased risk of stroke occurring with estrogen use 4. Patients with migraines may be prescribed estrogen without any concerns ____ 10. A 22-year-old woman receives a prescription for oral contraceptives. Education for this patient includes: 1. Counseling regarding decreasing or not smoking while taking oral contraceptives 2. Advising a monthly pregnancy test for the first 3 months she is taking the contraceptive 3. Advising that she may miss two pills in a row and not be concerned about pregnancy 4. Recommending that her next follow-up visit is in 1 year for a refill and annual exam ____ 11. A 19-year-old female is a nasal Staph aureus carrier and is placed on 5 days of rifampin for treatment. Her only other medication is combined oral contraceptives. What education should she receive regarding her medications? 1. Separate the oral ingestion of the rifampin and oral contraceptive by at least an hour. 2. Both medications are best tolerated if taken on an empty stomach. 3. She should use a back-up method of birth control such as condoms for the rest of the current pill pack. 4. If she gets nauseated with the medications she should call the office for an antiemetic prescription. ____ 12. A 56-year-old woman is complaining of vaginal dryness and dyspareunia. To treat her symptoms with the lowest adverse effects she should be prescribed: 1. Low-dose oral estrogen 2. A low-dose estrogen/progesterone combination 3. A vaginal estradiol ring 4. Vaginal progesterone cream ____ 13. Shana is receiving her first medroxyprogesterone (Depo Provera) injection. Shana will need to be monitored for: 1. Depression 2. Hypertension 3. Weight loss 4. Cataracts ____ 14. When prescribing medroxyprogesterone (Depo Provera) injections, essential education would include advising of the following potential adverse drug effects: 1. Hypertension and dysuria 2. Depression and weight gain 3. Abdominal pain and constipation 4. Orthostatic hypotension and dermatitis ____ 15. The medroxyprogesterone (Depo Provera) injection has a Black Box Warning due to: 1. The potential development of significant hypertension 2. Increased risk of strokes 3. Decreased bone density 4. The risk of a life-threatening rash such as Stevens-Johnson ____ 16. Shana received her first medroxyprogesterone (Depo Provera) injection 6 weeks ago and calls the clinic with a concern that she has been having a light “period” off and on since receiving her Depo shot. What would be the management of Shana? 1. Reassurance that some spotting is normal the first few months of Depo and it should improve. 2. Schedule an appointment for an exam as this is not normal. 3. Prescribe 4 weeks of estrogen to treat the abnormal vaginal bleeding. 4. Order a pregnancy test and suggest she use a back-up method of contraception until she has her next shot. ____ 17. William is a 62-year-old male who is requesting a prescription for sildenafil (Viagra). He should be screened for ________________ before prescribing sildenafil. 1. Renal dysfunction 2. Unstable coronary artery disease 3. Benign prostatic hypertrophy 4. History of priapism ____ 18. Men who are prescribed sildenafil (Viagra) need ongoing monitoring for: 1. Development of chest pain or dizziness 2. Weight gain 3. Priapism 4. Renal function ____ 19. Men who are prescribed an erectile dysfunction drug such as sildenafil (Viagra) should be warned about the risk for: 1. Impotence when combined with antihypertensives 2. Fatal hypotension if combined with nitrates 3. Weight gain if combined with antidepressants 4. All of the above ____ 20. Androgens are indicated for: 1. Symptomatic treatment for male deficiency 2. Female libido, endometriosis, and postmenopausal symptoms 3. Increased muscle mass 4. Symptomatic treatment in both sexes for cancer and HIV 5. 1, 2, and 4 6. All of the above ____ 21. Long-term use of androgens requires specific laboratory monitoring of: 1. Glucose, calcium, testosterone, and thyroid function 2. Calcium, testosterone, PSA, and liver function 3. Calcium, testosterone, PSA, liver function, glucose, and lipids 4. CBC, testosterone, PSA, and thyroid level ____ 22. Effects of estrogen include: 1. Regulation of the menstrual cycle 2. Maintenance of bone density by increasing bone reabsorption 3. Maintenance of the normal structure of the skin and blood vessels 4. A and C 5. All of the above ____ 23. Absolute contraindications that clinicians must consider when initiating estrogen therapy include: 1. Undiagnosed dysfunctional uterine bleeding 2. Deep vein or arterial thromboemboli within the prior year 3. Endometriosis 4. 1 and 2 5. All of the above ____ 24. Patients taking hormonal contraceptives and hormone replacement therapy need to take the drug daily at the same time to prevent: 1. Nausea 2. Breakthrough bleeding 3. Breast tenderness 4. Pregnancy Chapter 23. Drugs Affecting the Integumentary System ____ 1. Erik presents with one golden-crusted lesion at the site of an insect bite consistent with impetigo. His parents have limited finances and request the least expensive-treatment. Which medication would be the best choice for treatment? 1. Mupirocin (Bactroban) 2. Bacitracin and polymixin B (generic double antibiotic ointment) 3. Retapamulin (Altabax) 4. Oral cephalexin (Keflex) ____ 2. Juakeem is a nasal methicillin resistant staphylococcus aureus (MRSA) carrier. Treatment to eradicate nasal MRSA is mupirocin. Patient education regarding treating nasal MRSA includes: 1. Take the oral medication exactly as prescribed. 2. Insert one-half of the dose in each nostril twice a day. 3. Alternate treating one nare in the morning and the other in the evening. 4. Nasal MRSA eradication requires at least 4 weeks of therapy, with up to 8 weeks needed in some patients. ____ 3. Instructions for applying a topical antibiotic or antiviral ointment include: 1. Apply thickly to the infected area, spreading the medication well past the borders of the infection. 2. If the rash worsens, apply a thicker layer of medication to settle down the infection. 3. Wash hands before and after application of topical antimicrobials. 4. None of the above ____ 4. When Sam used clotrimazole (Lotrimin AF) for athlete’s foot he developed a red, itchy rash consistent with a hypersensitivity reaction. He now has athlete’s foot again. What would be a good choice of antifungal for Sam? 1. Miconazole (Micatin) powder 2. Ketoconazole (Nizoral) cream 3. Terbinafine (Lamisil) cream 4. Griseofulvin (Grifulvin V) suspension ____ 5. When prescribing griseofulvin (Grifulvin V) to treat tinea capitis it is critical to instruct the patient or parent to: 1. Mix the griseofulvin with ice cream before administering 2. Take the griseofulvin until the tinea clears, in approximately 4 to 5 weeks 3. Shampoo with baby shampoo daily while taking the griseofulvin 4. Griseofulvin is best absorbed if ingested with a high-fat food. ____ 6. First-line therapy for treating topical fungal infections such as tinea corporis (ringworm) or tinea pedis (athlete’s foot) would be: 1. OTC topical azole (clotrimazole, miconazole) 2. Oral terbinafine 3. Oral griseofulvin microsize 4. Nystatin cream or ointment ____ 7. When prescribing topical penciclovir (Denavir) for the treatment of herpes labialis (cold sores) patient education would include: 1. Spread penciclovir liberally all over lips and area surrounding lips. 2. Penciclovir therapy is started at the first sign of a cold sore outbreak. 3. Skin irritation is normal with penciclovir and it should resolve. 4. The penciclovir should be used a minimum of 2 weeks to prevent recurrence. ____ 8. Erika has been prescribed isotretinoin (Accutane) by her dermatologist and is presenting to her primary care provider with symptoms of sadness and depression. A Beck’s Depression Scale indicates she has mild to moderate depression. What would be the best care for her at this point? 1. Prescribe a select serotonin reuptake inhibitor (SSRI) antidepressant 2. Refer her to a mental health therapist 3. Contact her dermatologist about discontinuing the isotretinoin 4. Reassure her that mood swings are normal and schedule follow up in a week ____ 9. Drew is a 17-year-old competitive runner who presents with complaint of pain in his hip that occurred after he fell while running. His only medical problem is severe acne for which he takes isotretinoin (Accutane). With this history what would you be concerned for? 1. He may have pulled a muscle and needs to rest to recover. 2. He is at risk for bone injuries and needs to be evaluated for fracture. 3. Isotretinoin interacts with ibuprofen which is the pain medication of choice. 4. Teen athletes are at risk for repetitive stress injuries. ____ 10. Catherine calls the clinic with concerns that her acne is worse 1 week after starting topical tretinoin. What would be the appropriate care for her? 1. Change her to a different topical acne medication as she is having an adverse reaction to the tretinoin. 2. Switch her to an oral antibiotic to treat her acne. 3. Advise her to apply an oil-based lotion to her face to soothe the redness. 4. Reassure her that the worsening of acne is normal and it should improve with continued use. ____ 11. Li is a 6-month-old infant with severe eczema. She would benefit from topical corticosteroid therapy. Instructions for using topical corticosteroids in children include: 1. Apply liberally to all areas with eczema. 2. Double the frequency of application when the eczema is severe. 3. Apply sparingly to eczema areas. 4. Cover the eczema area with an occlusive dressing after applying a corticosteroid. ____ 12. Jose has had eczema for many years and reports that he thinks his corticosteroid cream is not working as well as it was previously. He may be experiencing tolerance to the corticosteroid. Treatment options include: 1. Increase the potency of the corticosteroid cream. 2. Recommend an interrupted or cyclic schedule of application. 3. Increase the frequency of dosing of the corticosteroid. 4. Discontinue the corticosteroid because it isn’t working any longer . ____ 13. When prescribing tacrolimus (Protopic) to treat atopic dermatitis patients should be informed that: 1. Tacrolimus is the most effective if it is used continuously for 4 to 6 months. 2. Tacrolimus should be spread generously over the affected area. 3. The FDA has issued a Black Box warning about the use of tacrolimus and the development of cancer in animals and humans. 4. The FDA recommends patients be screened for cancer before prescribing tacrolimus. ____ 14. Patients who are treated with greater than 100 grams per week of topical calcipotriene for psoriasis need to be monitored for: 1. High vitamin D levels 2. Hyperkalemia 3. Hypercalcemia 4. Hyperuricemia ____ 15. Jesse is prescribed tazarotene for his psoriasis. Patient education regarding topical tazarotene includes instructing them: 1. That tazarotene is applied in a thin film to the psoriasis plaque lesions 2. To apply it liberally to all psoriatic lesions 3. To apply tazarotene to nonaffected areas to prevent breakout 4. That tazarotene may cause hypercalcemia if it is overused ____ 16. Instructions for the use of selenium sulfide shampoo (Selsun) to treat scalp seborrhea include: 1. Shampoo daily and rinse well. 2. Worsening of seborrhea for the first week is normal. 3. Seborrhea usually clears up after a few weeks of treatment. 4. Shampoo twice a week for 2 weeks, then weekly. ____ 17. Topical diphenhydramine (Benadryl) is available OTC to treat itching. Patients or parents should be instructed regarding the use of topical diphenhydramine that: 1. For maximum effectiveness in treating itching, combine topical with oral diphenhydramine. 2. Topical diphenhydramine is the treatment of choice in treating poison ivy or poison oak. 3. Topical diphenhydramine should not be used in children younger than age 2 years. 4. When applying topical diphenhydramine, apply the cream liberally to all areas that itch. ____ 18. A patient has been prescribed silver sulfadiazine (Silvadene) cream to treat burns on his leg. Normal adverse effects of silver sulfadiazine cream include: 1. Transient leukopenia on days 2 to 4 that should resolve 2. Worsening of burn symptoms briefly before resolution 3. A red, scaly rash that will resolve with continued use 4. Hypercalcemia ____ 19. Instructions for the use of malathion (Ovide) for head lice include: 1. Use a blow dryer to dry the hair after applying. 2. Malathion (Ovide) is used daily for a week until all lice are dead. 3. Rinse the malathion (Ovide) off and shampoo hair after 8 to 12 hours. 4. Use gloves to apply the malathion (Ovide). ____ 20. When writing a prescription of permethrin 5% cream (Elimite) for scabies, patient education would include: 1. All members of the household and personal contacts should also be treated. 2. Infants should have permethrin applied from the neck down. 3. The permethrin is washed off after 10 to 20 minutes. 4. Permethrin is flammable and to avoid open flame while the medication is applied. Chapter 24. Drugs Used in Treating Infectious Diseases ____ 1. Factors that place a patient at risk of developing an antimicrobial-resistant organism include: 1. Age over 50 years 2. School attendance 3. Travel within the U.S. 4. Inappropriate use of antimicrobials ____ 2. Infants and young children are at higher risk of developing antibiotic-resistant infections due to: 1. Developmental differences in pharmacokinetics of the antibiotics in children 2. The fact that children this age are more likely to be in daycare and exposed to pathogens from other children 3. Parents of young children insisting on preventive antibiotics so they don’t miss work when their child is sick 4. Immunosuppression from the multiple vaccines they receive in the first 2 years of life ____ 3. Providers should use an antibiogram when prescribing. An antibiogram is: 1. The other name for the Centers for Disease Control guidelines for prescribing antibiotics 2. An algorithm used for prescribing antibiotics for certain infections 3. The reference also known as the Pink Book, published by the Centers for Disease Control 4. A chart of the local resistance patterns to antibiotics developed by laboratories ____ 4. There is often cross-sensitivity and cross-resistance between penicillins and cephalosporins because: 1. Renal excretion is similar in both classes of drugs. 2. When these drug classes are metabolized in the liver they both produce resistant enzymes. 3. Both drug classes contain a beta-lactam ring that is vulnerable to beta-lactamase-producing organisms. 4. There is not an issue with cross-resistance between the penicillins and cephalosporins. ____ 5. Jonathan has been diagnosed with strep throat and needs a prescription for an antibiotic. He says the last time he had penicillin he developed a red, blotchy rash. An appropriate antibiotic to prescribe would be: 1. Penicillin VK, because his rash does not sound like a serious rash 2. Amoxicillin 3. Cefadroxil (Duricef) 4. Azithromycin ____ 6. Sarah is a 25-year-old female who is 8 weeks pregnant and has a urinary tract infection. What would be the appropriate antibiotic to prescribe for her? 1. Ciprofloxacin (Cipro) 2. Amoxicillin (Trimox) 3. Doxycycline 4. Trimethoprim-sulfamethoxazole (Septra) ____ 7. Pong-tai is a 12-month-old child who is being treated with amoxicillin for acute otitis media. His parents call the clinic and say he has developed diarrhea. The appropriate action would be to: 1. Advise the parents that some diarrhea is normal with amoxicillin and recommend probiotics daily. 2. Change the antibiotic to one that is less of a gastrointestinal irritant. 3. Order stool cultures for suspected viral pathogens not treated by the amoxicillin. 4. Recommend increased fluids and fiber in his diet. ____ 8. Lauren is a 13-year-old child who comes to clinic with a 4-day history of cough, low-grade fever, and rhinorrhea. When she blows her nose or coughs the mucous is greenish-yellow. The appropriate antibiotic to prescribe would be: 1. Amoxicillin 2. Amoxicillin/clavulanate 3. TMP/SMZ (Septra) 4. None ____ 9. Joanna had a small ventricle septal defect (VSD) repaired when she was 3 years old and has no residual cardiac problems. She is now 28 and is requesting prophylactic antibiotics for an upcoming dental visit. The appropriate antibiotic to prescribe according to current American College of Cardiology and American Heart Association guidelines is: 1. None, no antibiotic is required for dental procedures 2. Amoxicillin 2 grams 1 hour before the procedure 3. Ampicillin 2 grams IM or IV 30 minutes before the procedure 4. Azithromycin 1 gram 1 hour before the procedure ____ 10. To prevent further development of antibacterial resistance it is recommended that fluoroquinolones be reserved for treatment of: 1. Urinary tract infections in young women 2. Upper respiratory infections in adults 3. Skin and soft tissue infections in adults 4. Community-acquired pneumonia in patients with comorbidities ____ 11. Fluoroquinolones have a Black Box Warning regarding ________ even months after treatment. 1. Renal dysfunction 2. Hepatic toxicity 3. Tendon rupture 4. Development of glaucoma ____ 12. Janet was recently treated with clindamycin for an infection. She calls the advice nurse because she is having frequent diarrhea that she thinks may have blood in it. What would be the appropriate care for her? 1. Encourage increased fluids and fiber. 2. Assess her for pseudomembranous colitis. 3. Advise her to eat yogurt daily to help restore her gut bacteria. 4. Start her on an antidiarrheal medication. ____ 13. Keng has chronic hepatitis that has led to mildly impaired liver function. He has an infection that would be best treated by a macrolide. Which would be the best choice for a patient with liver dysfunction? 1. Azithromycin (Zithromax) 2. Clarithromycin (Biaxin) 3. Erythromycin (E-mycin) 4. None of the above ____ 14. Jamie has glucose-6-phosphate dehydrogenase deficiency (G6PD) and requires an antibiotic. Which class of antibiotics should be avoided in this patient? 1. Penicillins 2. Macrolides 3. Cephalosporins 4. Sulfonamides ____ 15. If a patient is allergic to sulfonamide antibiotics, he or she will most likely have cross-sensitivity to: 1. Loop diuretics 2. Sulfonylureas 3. Thiazide diuretics 4. All of the above ____ 16. Tetracyclines such as minocycline are safe to use in: 1. Pregnant women 2. Adolescents 3. Patients with renal dysfunction 4. Patients with hepatic dysfunction ____ 17. Tetracyclines should not be prescribed to children younger than 8 years due to: 1. Risk of developing cartilage problems 2. Development of significant diarrhea 3. Risk of kernicterus 4. Adverse effects on bone growth ____ 18. Nicole is a 16-year-old female who is taking minocycline for acne. She comes to the clinic complaining of a headache. What would be the plan of care? 1. Advise acetaminophen or ibuprofen as needed for headaches. 2. Prescribe sumatriptan (Imitrex) to be taken at the onset of the headache. 3. Evaluate her for pseudotremor cerebri. 4. Assess her caffeine intake and sleep patterns. ____ 19. Patricia has been prescribed doxycycline for a chlamydia infection. She is healthy and her only medication is an oral combined contraceptive. Patricia’s education would include: 1. Use a back-up method of birth control (condom) until her next menses. 2. Doxycycline may cause tendonitis and she should report any joint pain. 3. Her partner will need treatment if her infection doesn’t clear with the doxycycline. 4. Doxycycline is used for one-dose treatment of STIs; take the whole prescription at once. ____ 20. To prevent the development of peripheral neuropathy in patients taking isoniazid for tuberculosis the patient is also prescribed: 1. Niacin (vitamin B3) 2. Pyridoxine (vitamin B6) 3. Riboflavin (vitamin B2) 4. Thiamine (vitamin B1) ____ 21. Sadie is an 82-year-old patient who has herpes zoster (shingles) and would benefit from an antiviral such as valacyclovir. Prior to prescribing valacyclovir she will need an assessment of: 1. Complete blood count to rule out anemia 2. Liver function 3. Renal function 4. Immunocompetence ____ 22. When prescribing acyclovir, patients should be educated regarding the: 1. High risk of developing diarrhea 2. Need to drink lots of fluids during treatment 3. Risk for life-threatening rash such as Stevens-Johnson 4. Eccentric dosing schedule ____ 23. Nicholas has been diagnosed with type A influenza. Appropriate prescribing of oseltamivir (Tamiflu) would include: 1. Starting oseltamivir within the first 48 hours of influenza symptoms 2. Advising the patient he can stop the oseltamivir when his symptoms resolve 3. Educating the patient that oseltamivir will cure influenza 4. Prophylactic treatment of all family members ____ 24. Monitoring for patients who are on long-term antifungal therapy with ketoconazole includes: 1. Platelet count 2. BUN and creatinine 3. White blood cell count 4. AST, ALT, alkaline phosphatase, and bilirubin ____ 25. When prescribing metronidazole (Flagyl) to treat bacterial vaginosis, patient education would include: 1. Metronidazole is safe in the first trimester of pregnancy. 2. Consuming alcohol in any form may cause a severe reaction. 3. Sexual partners need concurrent therapy. 4. Headaches are a sign of a serious adverse reaction and need immediate evaluation. ____ 26. Every antibiotic drug class has resistant organisms that influence prescribing decisions. 1. True 2. False Chapter 25. Drugs Used in Treating Inflammatory Processes ____ 1. Henry presents to clinic with a significantly swollen, painful great toe and is diagnosed with gout. Of the following, which would be the best treatment for Henry? 1. High-dose colchicine 2. Low-dose colchicine 3. High-dose aspirin 4. Acetaminophen with codeine ____ 2. Patient education when prescribing colchicine includes: 1. Colchicine may be constipating. 2. Colchicine always causes some degree of diarrhea. 3. Mild muscle weakness is normal. 4. Moderate amounts of alcohol are safe with colchicine. ____ 3. Larry is taking allopurinol to prevent gout. Monitoring of a patient who is taking allopurinol includes: 1. Complete blood count 2. Blood glucose 3. C-reactive protein 4. BUN, creatinine, and creatinine clearance ____ 4. Phil is starting treatment with febuxostat (Uloric). Education of patients starting febuxostat includes: 1. Gout may worsen with therapy. 2. Febuxostat may cause severe diarrhea. 3. He should consume a high-calcium diet. 4. He will need frequent CBC monitoring. ____ 5. Sallie has been taking 10 mg per day of prednisone for the past 6 months. She should be assessed for: 1. Gout 2. Iron deficiency anemia 3. Osteoporosis 4. Renal dysfunction ____ 6. Patients whose total dose of prednisone will exceed 1 gram will most likely need a second prescription for: 1. Metformin, a biguanide to prevent diabetes 2. Omeprazole, a proton pump inhibitor to prevent peptic ulcer disease 3. Naproxen, an NSAID to treat joint pain 4. Furosemide, a diuretic to treat fluid retention ____ 7. Daniel has been on 60 mg of prednisone for 10 days to treat a severe asthma exacerbation. It is time to discontinue the prednisone. How is prednisone discontinued? 1. Patients with asthma are transitioned directly off the prednisone onto inhaled corticosteroids. 2. Prednisone can be abruptly discontinued with no adverse effects. 3. Develop a tapering schedule to slowly wean Daniel off the prednisone. 4. Substitute the prednisone with another anti-inflammatory such as ibuprofen. ____ 8. Patients with rheumatoid arthritis who are on chronic low-dose prednisone will need co-treatment with which medications to prevent further adverse effects? 1. A bisphosphonate 2. Calcium supplementation 3. Vitamin D 4. All of the above ____ 9. Patients who are on or who will be starting chronic corticosteroid therapy need monitoring of: 1. Serum glucose 2. Stool culture 3. Folate levels 4. Vitamin B12 ____ 10. Patients who are on chronic long-term corticosteroid therapy need education regarding: 1. Receiving all vaccinations, especially the live flu vaccine 2. Reporting black tarry stools or abdominal pain 3. Eating a high carbohydrate diet with plenty of fluids 4. Small amounts of alcohol are generally tolerated. ____ 11. All nonsteroidal anti-inflammatory drugs (NSAIDS) have an FDA Black Box Warning regarding: 1. Potential for causing life-threatening GI bleeds 2. Increased risk of developing systemic arthritis with prolonged use 3. Risk of life-threatening rashes, including Stevens-Johnson 4. Potential for transient changes in serum glucose ____ 12. Jamie has fractured his ankle and has received a prescription for acetaminophen and hydrocodone (Vicodin). Education when prescribing Vicodin includes: 1. It is okay to double the dose of Vicodin if the pain is severe. 2. Vicodin is not habit-forming. 3. He should not take any other acetaminophen-containing medications. 4. Vicodin may cause diarrhea; increase his fluid intake. ____ 13. When prescribing NSAIDS, a complete drug history should be conducted as NSAIDs interact with these drugs: 1. Omeprazole, a proton pump inhibitor 2. Combined oral contraceptives 3. Diphenhydramine, an antihistamine 4. Warfarin, an anticoagulant ____ 14. Josefina is a 2-year-old child with acute otitis media and an upper respiratory infection. Along with an antibiotic she receives a recommendation to treat the ear pain with ibuprofen. What education would her parent need regarding ibuprofen? 1. They can cut an adult ibuprofen tablet in half to give Josefina. 2. The ibuprofen dose can be doubled for severe pain. 3. Josefina needs to be well-hydrated while taking ibuprofen. 4. Ibuprofen is completely safe in children with no known adverse effects. ____ 15. Henry is 82 years old and takes two aspirin every morning to treat the arthritis pain in his back. He states the aspirin helps him to “get going” each day. Lately he has had some heartburn from the aspirin. After ruling out an acute GI bleed, what would be an appropriate course of treatment for Henry? 1. Add an H2 blocker such as ranitidine to his therapy. 2. Discontinue the aspirin and switch him to Vicodin for the pain. 3. Decrease the aspirin dose to one tablet daily. 4. Have Henry take an antacid 15 minutes before taking the aspirin each day. ____ 16. The trial period to determine effective anti-inflammatory activity when starting a patient on aspirin for rheumatoid arthritis is: 1. 48 hours 2. 4 to 6 days 3. 4 weeks 4. 2 months ____ 17. Patients prescribed aspirin therapy require education regarding the signs of aspirin toxicity. An early sign of aspirin toxicity is: 1. Black tarry stools 2. Vomiting 3. Tremors 4. Tinnitus ____ 18. Monitoring a patient on a high-dose aspirin level includes: 1. Salicylate level 2. Complete blood count 3. Urine pH 4. All of the above ____ 19. Patients who are on long-term aspirin therapy should have ______ annually. 1. Complete blood count 2. Salicylate level 3. Amylase 4. Urine analysis Chapter 27. Anemia ____ 1. Pernicious anemia is treated with: 1. Folic acid supplements 2. Thiamine supplements 3. Vitamin B12 4. Iron ____ 2. Premature infants require iron supplementation with: 1. 10 mg/day of iron 2. 2 mg/kg per day until age 12 months 3. 7 mg/day in their diet 4. 1 mg/kg per day until they are receiving adequate intake of iron from foods ____ 3. Breastfed infants should receive iron supplementation of: 1. 3 mg/kg per day 2. 6 mg/kg per day 3. 1 mg/kg per day 4. Breastfed babies do not need iron supplementation ____ 4. Valerie presents to the clinic with menorrhagia. Her hemoglobin is 10.2 and her ferritin is 15 ng/mL. Initial treatment for her anemia would be: 1. 18 mg/day of iron supplementation 2. 6 mg/kg per day of iron supplementation 3. 325 mg ferrous sulfate per day 4. 325 mg ferrous sulfate tid ____ 5. Chee is a 15-month-old male whose screening hemoglobin is 10.4 g/dL. Treatment for his anemia would be: 1. 18 mg/day of iron supplementation 2. 6 mg/kg per day of elemental iron 3. 325 mg ferrous sulfate per day 4. 325 mg ferrous sulfate tid ____ 6. Monitoring for a patient taking iron to treat iron deficiency anemia is: 1. Hemoglobin, hematocrit, and ferritin 4 weeks after treatment is started 2. Complete blood count every 4 weeks throughout treatment 3. Annual complete blood count 4. Reticulocyte count in 4 weeks ____ 7. Valerie has been prescribed iron to treat her anemia. Education of patients prescribed iron would include: 1. Take the iron with milk if it upsets her stomach. 2. Antacids may help with the nausea and GI upset caused by iron. 3. Increase fluids and fiber to treat constipation. 4. Iron is best tolerated if it is taken at the same time as her other medications. ____ 8. Allie has just had her pregnancy confirmed and is asking about how to ensure a healthy baby. What is the folic acid requirement during pregnancy? 1. 40 mcg/day 2. 200 mcg/day 3. 800 mcg/day 4. 2 gm/day ____ 9. Kyle has Crohn’s disease and has a documented folate deficiency. Drug therapy for folate deficiency anemia is: 1. Oral folic acid 1 to 2 mg per day 2. Oral folic acid 1 gram per day 3. IM folate weekly for at least 6 months 4. Oral folic acid 400 mcg daily ____ 10. Patients who are being treated for folate deficiency require monitoring of: 1. Complete blood count every 4 weeks 2. Hematocrit and hemoglobin at 1 week and then at 8 weeks 3. Reticulocyte count at 1 week 4. Folate levels every 4 weeks until hemoglobin stabilizes ____ 11. The treatment of vitamin B12 deficiency is: 1. 1,000 mcg daily of oral cobalamin 2. 2 gm per day of oral cobalamin 3. Vitamin B12 100 mcg/day IM 4. 500 mcg/dose nasal cyanocobalamin 2 sprays once a week ____ 12. The dosage of Vitamin B12 to initially treat pernicious anemia is: 1. Nasal cyanocobalamin 1 gram spray in each nostril daily x 1 week then weekly x 1 month 2. Vitamin B12 IM monthly 3. Vitamin B12 1,000 mcg IM daily x 1 week then 1,000 mg IM weekly for a month 4. Oral cobalamin 1,000 mcg daily ____ 13. Before beginning IM vitamin B12 therapy, which laboratory values should be obtained? 1. Reticulocyte count, hemoglobin, and hematocrit 2. Iron 3. Vitamin B12 4. All of the above ____ 14. ________ should be monitored when vitamin B12 therapy is started. 1. Serum calcium 2. Serum potassium 3. Ferritin 4. C-reactive protein ____ 15. Anemia due to chronic renal failure is treated with: 1. Epoetin alfa (Epogen) 2. Ferrous sulfate 3. Vitamin B12 4. Hydroxyurea Chapter 28. Chronic Stable Angina and Low-Risk Unstable Angina 1. Angina is produced by an imbalance between myocardial oxygen supply (MOS) and demand (MOD) in the myocardium. Which of the following drugs help to correct this imbalance by increasing MOS? 1. Calcium channel blockers 2. Beta blockers 3. Angiotensin-converting-enzyme (ACE) inhibitors 4. Aspirin 2. Not all chest pain is caused by myocardial ischemia. Noncardiac causes of chest pain include: 1. Pulmonary embolism 2. Pneumonia 3. Gastroesophageal reflux 4. All of the above 3. The New York Heart Association and the Canadian Cardiovascular Society have described grading criteria for levels of angina. Angina that occurs with unusually strenuous activity or on walking or climbing stair after meals is class: 1. I 2. II 3. III 4. IV 4. Patients at high risk for developing significant coronary heart disease are those with: 1. LDL values between 100 and 130 2. Systolic blood pressure between 120 and 130 3. Class III angina 4. Obesity 5. To reduce mortality, all patients with angina, regardless of class, should be on: 1. Aspirin 81 to 325 mg/d 2. Nitroglycerin sublingually for chest pain 3. ACE inhibitors or angiotensin receptor blockers 4. Digoxin 6. Patients who have angina, regardless of class, who are also diabetic, should be on: 1. Nitrates 2. Beta blockers 3. ACE inhibitors 4. Calcium channel blockers 7. Management of all types and grades of angina includes the use of lifestyle modification to reduce risk factors. Which of these modifications are appropriate for which reason? Both the modification and the reason for it must be true for the answer to be correct. 1. Lose at least 10 pounds of body weight. Excessive weight increases cardiac workload. 2. Reduce sodium intake to no more than 2,400 mg of sodium. Sodium increases blood volume and cardiac workload. 3. Increase potassium intake to at least 100 mEq/d. The heart needs higher levels of potassium to improve contractility and oxygen supply. 4. Intake a moderate amount of alcohol. Moderate intake has been shown by research to improve cardiac function. 8. Nitrates are especially helpful for patients with angina who also have: 1. Heart failure 2. Hypertension 3. Both 1 and 2 4. Neither 1 nor 2 9. Beta blockers are especially helpful for patients with exertional angina who also have: 1. Arrhythmias 2. Hypothyroidism 3. Hyperlipidemia 4. Atherosclerosis 10. Rapid-acting nitrates are important for all angina patients. Which of the following are true statements about their use? 1. These drugs are useful for immediate symptom relief when the patient is certain it is angina. 2. The dose is one sublingual tablet or spray every 5 minutes until the chest pain goes away. 3. Take one nitroglycerine tablet or spray at the first sign of angina; repeat every 5 minutes for no more than two doses. If chest pain is still not relieved, call 911. 4. All of the above 11. Isosorbide dinitrate is a long-acting nitrate given twice daily. The schedule for administration is 7 a.m. and 2 p.m. because: 1. Long-acting forms have a higher risk for toxicity. 2. Orthostatic hypotension is a common adverse effect. 3. It must be taken with milk or food. 4. Nitrate tolerance can develop. 12. Combinations of a long-acting nitrate and a beta blocker are especially effective in treating angina because: 1. Nitrates increase MOS and beta blockers increase MOD. 2. Their additive effects permit lower doses of both drugs and their adverse reactions cancel each other out. 3. They address the pathology of patients with exertional angina who have fixed atherosclerotic coronary heart disease. 4. All of the above 13. Drug choices to treat angina in older adults differ from those of younger adults only in: 1. Consideration of risk factors for diseases associated with and increased in aging 2. The placement of drug therapy as a treatment choice before lifestyle changes are tried 3. The need for at least three drugs in the treatment regimen because of the complexity of angina in the older adult 4. Those with higher risk for silent myocardial infarction 14. Which of the following drugs has been associated with increased risk for myocardial infarction in women? 1. Aspirin 2. Beta blockers 3. Estrogen replacement 4. Lipid-lowering agents 15. Cost of antianginal drug therapy should be considered in drug selection because of all of the following EXCEPT: 1. Patients often require multiple drugs 2. A large number of angina patients are older adults on fixed incomes 3. Generic formulations may be cheaper but are rarely bioequivalent 4. Lack of drug selectivity may result in increased adverse reactions 16. Situations that suggest referral to a specialist is appropriate include: 1. When chronic stable angina becomes unpredictable in its characteristics and precipitating factors 2. When a post-myocardial infarction patient develops new-onset angina 3. When standard therapy is not successful in improving exercise tolerance or reducing the incidence of angina 4. All of the above 17. The rationale for prescribing calcium blockers for angina can be based on the need for: 1. Increased inotropic effect in the heart 2. Increasing peripheral perfusion 3. Keeping heart rates high enough to ensure perfusion of coronary arteries 4. Help with rate control 18. Medications are typically started for angina patients when: 1. The first permanent EKG changes occur 2. The start of class I or II symptoms 3. The events trigger a trip to the emergency department 4. When troponin levels become altered 19. The most common cause of angina is: 1. Vasospasm of the coronary arteries 2. Atherosclerosis 3. Platelet aggregation 4. Low systemic oxygen 20. Ranolazine is used in angina patients to: 1. Dilate plaque-filled arteries 2. Inhibit platelet aggregation 3. Restrict late sodium flow in the myocytes 4. Induce vasoconstriction in the periphery to open coronary vessels 21. When is aspirin (ASA) used in angina patients? 1. All angina patients should be taking ASA unless it is contraindicated for allergy or other medical reasons. Chapter 29. Anxiety and Depression ____ 1. Common mistakes practitioners make in treating anxiety disorders include: 1. Switching medications after an 8- to 12-week trial 2. Maximizing dosing of antianxiety medications 3. Encouraging exercise and relaxation therapy before starting medication 4. Thinking a partial response to medication is acceptable ____ 2. An appropriate first-line drug to try for mild to moderate generalized anxiety disorder would be: 1. Alprazolam (Xanax) 2. Diazepam (Valium) 3. Buspirone (Buspar) 4. Amitriptyline (Elavil) ____ 3. An appropriate drug to initially treat panic disorder is: 1. Alprazolam (Xanax) 2. Diazepam (Valium) 3. Buspirone (Buspar) 4. Amitriptyline (Elavil) ____ 4. Prior to starting antidepressants, patients should have laboratory testing to rule out: 1. Hypothyroidism 2. Anemia 3. Diabetes mellitus 4. Low estrogen levels ____ 5. David is a 34-year-old patient who is starting on paroxetine (Paxil) for depression. David’s education regarding his medication would include: 1. Paroxetine may cause intermittent diarrhea. 2. He may experience sexual dysfunction beginning a month after he starts therapy. 3. He may have constipation and he should increase fluids and fiber. 4. Paroxetine has a long half-life so he may occasionally skip a dose. ____ 6. Jamison has been prescribed citalopram (Celexa) to treat his depression. Education regarding how quickly selective serotonin reuptake inhibitor (SSRI) antidepressants work would be: 1. Appetite and concentration improve in the first 1 to 2 weeks. 2. Sleep should improve almost immediately upon starting citalopram. 3. Full response to the SSRI may take 2 to 4 months after he reaches the full therapeutic dose. 4. His dysphoric mood will improve in 1 to 2 weeks. ____ 7. An appropriate drug for the treatment of depression with anxiety would be: 1. Alprazolam (Xanax) 2. Escitalopram (Lexapro) 3. Buspirone (Buspar) 4. Amitriptyline (Elavil) ____ 8. An appropriate first-line drug for the treatment of depression with fatigue and low energy would be: 1. Venlafaxine (Effexor) 2. Escitalopram (Lexapro) 3. Buspirone (Buspar) 4. Amitriptyline (Elavil) ____ 9. The laboratory monitoring required when a patient is on a selective serotonin reuptake inhibitor is: 1. Complete blood count every 3 to 4 months 2. Therapeutic blood levels every 6 months after a steady state is achieved 3. Blood glucose every 3 to 4 months 4. There is no laboratory monitoring required ____ 10. Jaycee has been on escitalopram (Lexapro) for a year and is willing to try tapering off of the selective serotonin reuptake inhibitor. What is the initial dosage adjustment when starting a taper off antidepressants? 1. Change dose to every other day dosing for a week 2. Reduce dose by 50% for 3 to 4 days 3. Reduce dose by 50% every other day 4. Escitalopram (Lexapro) can be stopped abruptly due to its long half-life ____ 11. The longer-term Xanax patient comes in and states they need a higher dose of the medication. They deny any additional, new, or accelerating triggers of their anxiety. What is the probable reason? 1. They have become tolerant of the medication, which is characterized by the need for higher and higher doses. 2. They are a drug seeker. 3. They are suicidal. 4. They only need additional counseling on lifestyle modification. ____ 12. What “onset of action” symptoms should be reviewed with patients who have been newly prescribed a selective serotonin reuptake inhibitor? 1. They will have insomnia for a week. 2. They can feel a bit of nausea, but this resolves in a week. 3. They will have an “onset seizure” but this is considered normal. 4. They will no longer dream. ____ 13. Which of the following should not be taken with a selective serotonin reuptake inhibitor? 1. Aged blue cheese 2. Grapefruit 3. Alcohol 4. Green leafy vegetables ____ 14. Why is the consistency of taking paroxetine (Paxil) and never running out of medication more important than with most other selective serotonin reuptake inhibitors (SSRIs)? 1. It has a shorter half-life and withdrawal syndrome has a faster onset without taper. 2. It has the longest half-life and the withdrawal syndrome has a faster onset. 3. It is quasi-addictive in the dopaminergic reward system. 4. It is the most activating of SSRI medications and will cause the person to have sudden deep sadness. ____ 15. The patient shares with the provider that he is taking his Prozac at night before going to bed. What is the best response? 1. This is a good idea because this class of medications generally makes people sleepy. 2. Have you noticed that you are having more sleep issues since you started that? 3. This a good way to remember to take your daily medications because it is near your toothbrush. 4. This is a good plan because you can eat grapefruit if there is 8–12 hours difference in the time each are ingested. Chapter 30. Asthma and Chronic Obstructive Pulmonary Disease ____ 1. Prior to developing a plan for the treatment of asthma, the patient’s asthma should be classified according to the NHLBI Expert Panel 3 guidelines. In adults mild-persistent asthma is classified as asthma symptoms that occur: 1. Daily 2. Daily and limit physical activity 3. Less than twice a week 4. More than twice a week and less than once a day ____ 2. In children age 5 to 11 years mild-persistent asthma is diagnosed when asthma symptoms occur: 1. At nighttime one to two times a month 2. At nighttime three to four times a month 3. Less than twice a week 4. Daily ____ 3. One goal of asthma therapy outlined by the NHLBI Expert Panel 3 guidelines is: 1. Ability to use albuterol daily to control symptoms 2. Minimize exacerbations to once a month 3. Keep nighttime symptoms at a maximum of twice a week 4. Require infrequent use of beta 2 agonists (albuterol) for relief of symptoms ____ 4. A stepwise approach to the pharmacologic management of asthma: 1. Begins with determining the severity of asthma and assessing asthma control 2. Is used when asthma is severe and requires daily steroids 3. Allows for each provider to determine their personal approach to the care of asthmatic patients 4. Provides a framework for the management of severe asthmatics, but is not as helpful when patients have intermittent asthma ____ 5. Treatment for mild intermittent asthma is: 1. Daily inhaled medium-dose corticosteroids 2. Short-acting beta-2-agonists (albuterol) as needed 3. Long-acting beta-2-agonists every morning as a preventative 4. Montelukast (Singulair) daily ____ 6. The first-line therapy for mild-persistent asthma is: 1. High-dose montelukast 2. Theophylline 3. Low-dose inhaled corticosteroids 4. Long-acting beta-2-agonists ____ 7. Monitoring a patient with persistent asthma includes: 1. Monitoring how frequently the patient has an upper respiratory infection (URI) during treatment 2. Monthly in-office spirometry testing 3. Determining if the patient has increased use of his or her long-acting beta-2-agonist due to exacerbations 4. Evaluating the patient every 1 to 6 months to determine if the patient needs to step up or down in their therapy ____ 8. Asthma exacerbations at home are managed by the patient by: 1. Increasing frequency of beta-2-agonists and contacting their provider 2. Doubling inhaled corticosteroid doses 3. Increasing frequency of beta-2-agonists 4. Starting montelukast (Singulair) ____ 9. Patients who are at risk of a fatal asthma attack include patients: 1. With moderate persistent asthma 2. With a history of requiring intubation or ICU admission for asthma 3. Who are on daily inhaled corticosteroid therapy 4. Who are pregnant ____ 10. Pregnant patients with asthma may safely use ________ throughout their pregnancy. 1. Oral terbutaline 2. Prednisone 3. Inhaled corticosteroids (budesonide) 4. Montelukast (Singulair) ____ 11. One goal of asthma management in children is: 1. They independently manage their asthma 2. Participation in school and sports activities 3. No exacerbations 4. Minimal use of inhaled corticosteroids ____ 12. Medications used in the management of patients with chronic obstructive pulmonary disease (COPD) include: 1. Inhaled beta-2-agonists 2. Inhaled anticholinergics (ipratropium) 3. Inhaled corticosteroids 4. All of the above ____ 13. Patients with a COPD exacerbation may require: 1. Doubling of inhaled corticosteroid dose 2. Systemic corticosteroid burst 3. Continuous inhaled beta-2-agonists 4. Leukotriene therapy ____ 14. Patients with COPD require monitoring of: 1. Beta-2-agonist use 2. Serum electrolytes 3. Blood pressure 4. Neuropsychiatric effects of montelukast ____ 15. Education of patients with COPD who use inhaled corticosteroids includes: 1. Doubling the dose at the first sign of a URI 2. Using their inhaled corticosteroid first and then their bronchodilator 3. Rinsing their mouth after use 84. Abstaining from smoking for at least 30 minutes after using ____ 16. Education for patients who use an inhaled beta-agonist and an inhaled corticosteroid includes: 1. Use the inhaled corticosteroid first, followed by the inhaled beta-agonists. 2. Use the inhaled beta-agonist first, followed by the inhaled corticosteroid. 3. Increase fluid intake to 3 liters per day. 4. Avoid use of aspirin or ibuprofen while using inhaled medications. Chapter 31. Contraception ____ 1. Women who are taking an oral contraceptive containing the progesterone drospirenone may require monitoring of: 1. Hemoglobin 2. Serum calcium 3. White blood count 4. Serum potassium ____ 2. The mechanism of action of oral combined contraceptives that prevents pregnancy is: 1. Estrogen prevents the luteinizing hormone surge necessary for ovulation. 2. Progestins thicken cervical mucus and slow tubal motility. 3. Estrogen thins the endometrium making implantation difficult. 4. Progestin suppresses follicle stimulating hormone release. ____ 3. To improve actual effectiveness of oral contraceptives women should be educated regarding: 1. Use of a back-up method if they have vomiting or diarrhea during a pill packet 2. Doubling pills if they have diarrhea during the middle of a pill pack 3. The fact that they will have a normal menstrual cycle if they miss two pills 4. The fact that mid-cycle spotting is not normal and the provider should be contacted immediately ____ 4. A contraindication to the use of combined contraceptives is: 1. Adolescence (not approved for this age) 2. A history of clotting disorder 3. Recent pregnancy 4. Being overweight ____ 5. Obese women may have increased risk of failure with which contraceptive method? 1. Combined oral contraceptives 2. Progestin-only oral contraceptive pill 3. Injectable progestin 4. Combined topical patch ____ 6. Ashley comes to the clinic with a request for oral contraceptives. She has successfully used oral contraceptives before and has recently started dating a new boyfriend so would like to restart contraception. She denies recent intercourse and has a negative urine pregnancy test in the clinic. An appropriate plan of care would be: 1. Recommend she return to the clinic at the start of her next menses to get a Depo Provera shot. 2. Prescribe oral combined contraceptives and recommend she start them at the beginning of her next period and use a back-up method for the first 7 days. 3. Prescribe oral contraceptives and have her start them the same day as the visit with a back-up method used for the first 7 days. 4. Discuss the advantages of using the topical birth control patch and recommend she consider using the patch. ____ 7. When discussing with a patient the different start methods used for oral combined contraceptives, the advantage of a Sunday start over the other start methods is: 1. Immediate protection against pregnancy the first week of using the pill 2. No back-up method is needed when starting 3. Menses occur during the week 4. They can start the pill on the Sunday after the office visit ____ 8. The topical patch combined contraceptive (Ortho Evra) is: 1. Started on the first day of the menstrual cycle 2. Recommended for women over 200 pounds 3. Not as effective as oral combined contraceptives 4. Known to have more adverse effects, such as nausea, than the oral combined contraceptives ____ 9. Progesterone-only pills are recommended for women who: 1. Are breastfeeding 2. Have a history of migraine 3. Have a medical history that contradicts the use of estrogen 4. All of the above ____ 10. Women who are prescribed progestin-only contraception need education regarding which common adverse drug effects? 1. Increased migraine headaches 2. Increased risk of developing blood clots 3. Irregular vaginal bleeding for the first few months 4. Increased risk for hypercalcemia ____ 11. An advantage of using the NuvaRing vaginal ring for contraception is: 1. It does not require fitting and is easy to insert. 2. It is inserted once a week, eliminating the need to remember to take a daily pill. 3. Patients get a level of estrogen and progestin equal to combined oral contraceptives. 4. It also provides protection against vaginal infections. ____ 12. Oral emergency contraception (Plan B) is contraindicated in women who: 1. Had intercourse within the past 72 hours 2. May be pregnant 3. Are taking combined oral contraceptives 4. Are using a diaphragm Chapter 32. Dermatologic Conditions ____ 1. When choosing a topical corticosteroid cream to treat diaper dermatitis, the ideal medication would be: 1. Intermediate potency corticosteroid ointment (Kenalog) 2. A combination of a corticosteroid and an antifungal (Lotrisone) 3. A low-potency corticosteroid cream applied sparingly (hydrocortisone 1%) 4. A high-potency corticosteroid cream (Diprolene AF) ____ 2. Topical immunomodulators such as pimecrolimus (Elidel) or tacrolimus (Protopic) are used for: 1. Short-term or intermittent treatment of atopic dermatitis 2. Topical treatment of fungal infections (Candida) 3. Chronic, inflammatory seborrheic dermatitis 4. Recalcitrant nodular acne ____ 3. Long-term treatment of moderate atopic dermatitis includes: 1. Topical corticosteroids and emollients 2. Topical corticosteroids alone 3. Topical antipruritics 4. Oral corticosteroids for exacerbations of atopic dermatitis ____ 4. Severe contact dermatitis caused by poison ivy or poison oak exposure often requires treatment with: 1. Topical antipruritics 2. Oral corticosteroids for 2 to 3 weeks 3. Thickly applied topical intermediate-dose corticosteroids 4. Isolation of the patient to prevent spread of the dermatitis ____ 5. When a patient has contact dermatitis, wet dressings with Domeboro solution are used for: 1. Cleaning the weeping area of dermatitis 2. Bathing the patient to prevent infection 3. Relief of inflammation 4. Providing a barrier layer to protect the surrounding skin ____ 6. Appropriate initial treatment for psoriasis would be: 1. An immunomodulator (Protopic or Elidel) 2. Wet soaks with Burrow’s or Domeboro solution 3. Intermittent therapy with intermediate potency topical corticosteroids 4. Anthralin (Drithocreme) ____ 7. Patient education when prescribing the vitamin D3 derivative calcipotriene for psoriasis includes: 1. Apply thickly to affected psoriatic areas two to three times a day. 2. A maximum of 100 grams per week may be applied. 3. Do not use calcipotriene in combination with their topical corticosteroids. 4. Calcipotriene may be augmented with the use of coal tar products. ____ 8. Mild acne may be initially treated with: 1. Topical combined antibiotic 2. Minocycline 3. Topical retinoid 4. OTC benzoyl peroxide ____ 9. Tobie presents to the clinic with moderate acne. He has been using OTC benzoyl peroxide at home with minimal improvement. A topical antibiotic (clindamycin) and a topical retinoid adapalene (Differin) are prescribed. Education of Tobie would include: 1. He should see an improvement in his acne within the first 2 weeks of treatment. 2. If there is no response in a week, double the daily application of adapalene (Differin). 3. He may see an initial worsening of his acne that will improve in 6 to 8 weeks. 4. Adapalene may cause bleaching of clothing. ____ 10. Josie has severe cystic acne and is requesting treatment with Accutane. The appropriate treatment for her would be: 1. Order a pregnancy test and if it is negative prescribe the isotretinoin (Accutane). 2. Order Accutane after educating her on the adverse effects. 3. Recommend she try oral antibiotics (minocycline). 4. Refer her to a dermatologist for treatment. ____ 11. The most cost-effective treatment for two or three impetigo lesions on the face is: 1. Mupirocin ointment 2. Retapamulin (Altabax) ointment 3. Topical clindamycin solution 4. Oral amoxicillin/clavulanate (Augmentin) ____ 12. Dwayne has classic tinea capitis. Treatment for tinea on the scalp is: 1. Miconazole cream rubbed in well for 4 weeks 2. Oral griseofulvin for 6 to 8 weeks 3. Ketoconazole shampoo daily for 6 weeks 4. Ciclopirox cream daily for 4 weeks ____ 13. Nicolas is a football player who presents to the clinic with athlete’s foot. Patients with tinea pedis may be treated with: 1. OTC miconazole cream for 4 weeks 2. Oral ketoconazole for 6 weeks 3. Mupirocin ointment for 2 weeks 4. Nystatin cream for 2 weeks ____ 14. Jim presents with fungal infection of two of his toenails (onychomycosis). Treatment for fungal infections of the nail includes: 1. Miconazole cream 2. Ketoconazole cream 3. Oral griseofulvin 4. Mupirocin cream ____ 15. Scabies treatment for a 4-year-old child includes a prescription for: 1. Permethrin 5% cream applied from the neck down 2. Pyrethrin lotion 3. Lindane 1% shampoo 4. All of the above ____ 16. Vanessa has been diagnosed with scabies. Her education would include: 1. She should apply the scabies treatment cream for an hour and wash it off. 2. Scabies may need to be retreated in a week after initial treatment. 3. All members of the household and close personal contacts should be treated. 4. Malathion is flammable and she should take care until the solution dries. ____ 17. Catherine has head lice and her mother is asking about what products are available that are not neurotoxic. The only non-neurotoxin head lice treatment is: 1. Permethrin 1% (Nix) 2. Lindane shampoo 3. Malathion (Ovide) 4. Benzoyl alcohol (Ulesfia) ____ 18. Rick has male pattern baldness on the vertex of his head and has been using Rogaine for 2 months. He asks how effective minoxidil (Rogaine) is. Minoxidil: 1. Provides a permanent solution to male pattern baldness if used for at least 4 months 2. Will show results after 4 months of twice-a-day use 3. May not work for Rick’s type of baldness 4. Works better if he also uses hydrocortisone cream daily on his scalp 33. Diabetes Mellitus ____ 1. Type 1 diabetes results from autoimmune destruction of the beta cells. Eighty-five to 90% of type 1 diabetics have: 1. Autoantibodies to two tyrosine phosphatases 2. Mutation of the hepatic transcription factor on chromosome 12 3. A defective glucokinase molecule due to a defective gene on chromosome 7p 4. Mutation of the insulin promoter factor ____ 2. Type 2 diabetes is a complex disorder involving: 1. Absence of insulin production by the beta cells 2. A suboptimal response of insulin-sensitive tissues in the liver 3. Increased levels of glucagon-like peptide in the postprandial period 4. Too much fat uptake in the intestine ____ 3. Diagnostic criteria for diabetes include: 1. Fasting blood glucose greater than 140 mg/dl on two occasions 2. Postprandial blood glucose greater than 140 mg/dl 3. Fasting blood glucose 100 to 125 mg/dl on two occasions 4. Symptoms of diabetes plus a casual blood glucose greater than 200 mg/dl ____ 4. Routine screening of asymptomatic adults for diabetes is appropriate for: 1. Individuals who are older than 45 and have a BMI of less than 25 kg/m2 2. Native Americans, African Americans, and Hispanics 3. Persons with HDL cholesterol greater than 100 mg/dl 4. Persons with prediabetes confirmed on at least two occasions ____ 5. Screening for children who meet the following criteria should begin at age 10 and occur every 3 years thereafter: 1. BMI above the 85th percentile for age and sex 2. Family history of diabetes in first- or second-degree relative 3. Hypertension based on criteria for children 4. Any of the above ____ 6. Insulin is used to treat both types of diabetes. It acts by: 1. Increasing beta cell response to low blood-glucose levels 2. Stimulating hepatic glucose production 3. Increasing peripheral glucose uptake by skeletal muscle and fat 4. Improving the circulation of free fatty acids ____ 7. Adam has type 1 diabetes and plays tennis for his university. He exhibits a knowledge deficit about his insulin and his diagnosis. He should be taught that: 1. He should increase his carbohydrate intake during times of exercise. 2. Each brand of insulin is equal in bioavailability, so buy the least expensive. 3. Alcohol produces hypoglycemia and can help control his diabetes when taken in small amounts. 4. If he does not want to learn to give himself injections, he may substitute an oral hypoglycemic to control his diabetes. ____ 8. Insulin preparations are divided into categories based on onset, duration, and intensity of action following subcutaneous injection. Which of the following insulin preparations has the shortest onset and duration of action? 1. Lispro 2. Glulisine 3. Glargine 4. Detemir ____ 9. The drug of choice for type 2 diabetics is metformin. Metformin: 1. Decreases glycogenolysis by the liver 2. Increases the release of insulin from beta cells 3. Increases intestinal uptake of glucose 4. Prevents weight gain associated with hyperglycemia ____ 10. Before prescribing metformin, the provider should: 1. Draw a serum creatinine level to assess renal function. 2. Try the patient on insulin. 3. Prescribe a thyroid preparation if the patient needs to lose weight. 4. All of the above ____ 11. Sulfonylureas may be added to a treatment regimen for type 2 diabetics when lifestyle modifications and metformin are insufficient to achieve target glucose levels. Sulfonylureas have been moved to Step 2 therapy because they: 1. Increase endogenous insulin secretion 2. Have a significant risk for hypoglycemia 3. Address the insulin resistance found in type 2 diabetics 4. Improve insulin binding to receptors ____ 12. Dipeptidyl peptidase-4 inhibitors (gliptins) act on the incretin system to improve glycemic control. Advantages of these drugs include: 1. Better reduction in glucose levels than other classes 2. Less weight gain than sulfonylureas 3. Low risk for hypoglycemia 4. Can be given twice daily ____ 13. Control targets for patients with diabetes include: 1. HbA1C between 7 and 8 2. Fasting blood glucose levels between 100 and 120 mg/dl 3. Blood pressure less than 130/80 mm Hg 4. LDL lipids less than 130 mg/dl ____ 14. Establishing glycemic targets is the first step in treatment of both types of diabetes. For type 1 diabetes: 1. Tight control/intensive therapy can be given to adults who are willing to test their blood glucose at least twice daily. 2. Tight control is acceptable for older adults if they are without complications. 3. Plasma glucose levels are the same for children as adults. 4. Conventional therapy has a fasting plasma glucose target between 120 and 150 mg/dl. ____ 15. Treatment with insulin for type 1 diabetics: 1. Starts with a total daily dose of 0.2 to 0.4 units per kg of body weight 2. Divides the total doses into three injections based on meal size 3. Uses a total daily dose of insulin glargine given once daily with no other insulin required 4. Is based on the level of blood glucose ____ 16. When the total daily insulin dose is split and given twice daily, which of the following rules may be followed? 1. Give two-thirds of the total dose in the morning and one-third in the evening. 2. Give 0.3 units per kg of premixed 70/30 insulin with one-third in the morning and two-thirds in the evening. 3. Give 50% of an insulin glargine dose in the morning and 50% in the evening. 4. Give long-acting insulin in the morning and short-acting insulin at bedtime. ____ 17. Studies have shown that control targets that reduce the HbA1C to less than 7% are associated with fewer long-term complications of diabetes. Patients who should have such a target include: 1. Those with long-standing diabetes 2. Older adults 3. Those with no significant cardiovascular disease 4. Young children who are early in their disease ____ 18. Prevention of conversion from prediabetes to diabetes in young children must take highest priority and should focus on: 1. Aggressive dietary manipulation to prevent obesity 2. Fostering LDL levels less than 100 mg/dl and total cholesterol less than 170 mg/dl to prevent cardiovascular disease 3. Maintaining a blood pressure that is less than 80% based on weight and height to prevent hypertension 4. All of the above ____ 19. The drugs recommended by the American Academy of Pediatrics for use in children with diabetes (depending upon type of diabetes) are: 1. Metformin and insulin 2. Sulfonylureas and insulin glargine 3. Split-mixed dose insulin and GPL-1 agonists 4. Biguanides and insulin lispro ____ 20. Unlike most type 2 diabetics where obesity is a major issue, older adults with low body weight have higher risks for morbidity and mortality. The most reliable indicator of poor nutritional status in older adults is: 1. Weight loss in previously overweight persons 2. Involuntary loss of 10% of body weight in less than 6 months 3. Decline in lean body mass over a 12-month period 4. Increase in central versus peripheral body adiposity ____ 21. The drugs recommended for older adults with type 2 diabetes include: 1. Second-generation sulfonylureas 2. Metformin 3. Pioglitazone 4. Third-generation sulfonylureas ____ 22. Ethnic groups differ in their risk for and presentation of diabetes. Hispanics: 1. Have a high incidence of obesity, elevated triglycerides, and hypertension 2. Do best with drugs that foster weight loss, such as metformin 3. Both 1 and 2 4. Neither 1 nor 2 ____ 23. The American Heart Association states that people with diabetes have a 2- to 4-fold increase in the risk of dying from cardiovascular disease. Treatments and targets that do not appear to decrease risk for micro- and macro-vascular complications include: 1. Glycemic targets between 7% and 7.5% 2. Use of insulin in type 2 diabetics 3. Control of hypertension and hyperlipidemia 4. Stopping smoking ____ 24. All diabetic patients with known cardiovascular disease should be treated with: 1. Beta blockers to prevent MIs 2. Angiotensin-converting enzyme inhibitors and aspirin to reduce risk of cardiovascular events 3. Sulfonylureas to decrease cardiovascular mortality 4. Pioglitazone to decrease atherosclerotic plaque buildup ____ 25. All diabetic patients with hyperlipidemia should be treated with: 1. HMG-CoA reductase inhibitors 2. Fibric acid derivatives 3. Nicotinic acid 4. Colestipol ____ 26. Both angiotensin converting enzyme inhibitors and some angiotensin II receptor blockers have been approved in treating: 1. Hypertension in diabetic patients 2. Diabetic nephropathy 3. Both 1 and 2 4. Neither 1 nor 2 ____ 27. Protein restriction helps slow the progression of albuminuria, glomerular filtration rate, decline, and end stage renal disease in some patients with diabetes. It is useful for patients who: 1. Cannot tolerate angiotensin converting enzyme inhibitors or angiotensin receptor blockers 2. Have uncontrolled hypertension 3. Have HbA1C levels above 7% 4. Show progression of diabetic nephropathy despite optimal glucose and blood pressure control ____ 28. Diabetic autonomic neuropathy (DAN) is the earliest and most common complication of diabetes. Symptoms associated with DAN include: 1. Resting tachycardia, exercise intolerance, and orthostatic hypotension 2. Gastroparesis, cold intolerance, and moist skin 3. Hyperglycemia, erectile dysfunction, and deficiency of free fatty acids 4. Pain, loss of sensation, and muscle weakness ____ 29. Drugs used to treat diabetic peripheral neuropathy include: 1. Metoclopramide 2. Cholinergic agonists 3. Cardioselective beta blockers 4. Gabapentin ____ 30. The American Diabetic Association has recommended which of the following tests for ongoing management of diabetes? 1. Fasting blood glucose 2. HbA1C 3. Thyroid function tests 4. Electrocardiograms ____ 31. Allison is an 18-year-old college student with type 1 diabetes. She is on NPH twice daily and Novolog before meals. She usually walks for 40 minutes each evening as part of her exercise regimen. She is beginning a 30-minute swimming class three times a week at 1 p.m. What is important for her to do with this change in routine? 1. Delay eating the midday meal until after the swimming class. 2. Increase the morning dose of NPH insulin on days of the swimming class. 3. Adjust the morning insulin injection so that the peak occurs while swimming. 4. Check glucose level before, during, and after swimming. ____ 32. Allison is an 18-year-old college student with type 1 diabetes. Allison’s pre-meal BG at 11:30 a.m. is 130. She eats an apple and has a sugar-free soft drink. At 1 p.m. before swimming her BG is 80. What should she do? 1. Proceed with the swimming class. 2. Recheck her BG immediately. 3. Eat a granola bar or other snack with CHO. 4. Take an additional dose of insulin. ____ 33. Bart is a patient is a 67-year-old male with T2 DM. He is on glipizide and metformin. He presents to the clinic with confusion, sluggishness, and extreme thirst. His wife tells you Bart does not follow his meal plan or exercise regularly, and hasn’t checked his BG for 1 week. A random glucose is drawn and it is 500. What is a likely diagnosis based on preliminary assessment? 1. Diabetic keto acidosis (DKA) 2. Hyperglycemic hyperosmolar syndrome (HHS) 3. Infection 4. Hypoglycemia ____ 34. What would one expected assessment finding be for hyperglycemic hyperosmolar syndrome? 1. Low hemoglobin 2. Ketones in the urine 3. Deep, labored breathing 4. pH of 7.35 ____ 35. A patient on metformin and glipizide arrives at her 11:30 a.m. clinic appointment diaphoretic and dizzy. She reports taking her medication this morning and ate a bagel and coffee for breakfast. BP is 110/70 and random finger-stick glucose is 64. How should this patient be treated? 1. 12 oz apple juice with 1 tsp sugar 2. 10 oz diet soda 3. 8 oz milk or 4 oz orange juice 4. 4 cookies and 8 oz chocolate milk Chapter 34. Gastroesophageal Reflux and Peptic Ulcer Disease ____ 1. Gastroesophageal reflux disease may be aggravated by the following medication that affects lower esophageal sphincter (LES) tone: 1. Calcium carbonate 2. Estrogen 3. Furosemide 4. Metoclopramide ____ 2. Lifestyle changes are the first step in treatment of gastroesophageal reflux disease (GERD). Food or drink that may aggravate GERD include: 1. Eggs 2. Caffeine 3. Chocolate 4. Soda pop ____ 3. Metoclopramide improves gastroesophageal reflux disease symptoms by: 1. Reducing acid secretion 2. Increasing gastric pH 3. Increasing lower esophageal tone 4. Decreasing lower esophageal tone ____ 4. Antacids treat gastroesophageal reflux disease by: 1. Increasing lower esophageal tone 2. Increasing gastric pH 3. Inhibiting gastric acid secretion 4. Increasing serum calcium level ____ 5. When treating patients using the “Step-Down” approach the patient with gastroesophageal reflux disease is started on _______ first. 1. Antacids 2. Histamine2 receptor antagonists 3. Prokinetics 4. Proton pump inhibitors ____ 6. If a patient with symptoms of gastroesophageal reflux disease states that he has been self-treating at home with OTC ranitidine daily, the appropriate treatment would be: 1. Prokinetic (metoclopramide) for 4 to 8 weeks 2. Proton pump inhibitor (omeprazole) for 12 weeks 3. Histamine2 receptor antagonist (ranitidine) for 4 to 8 weeks 4. Cytoprotective drug (misoprostol) for 2 weeks ____ 7. If a patient with gastroesophageal reflux disease who is taking a proton pump inhibitor daily is not improving, the plan of care would be: 1. Prokinetic (metoclopramide) for 8 to 12 weeks 2. Proton pump inhibitor (omeprazole) twice a day for 4 to 8 weeks 3. Histamine2 receptor antagonist (ranitidine) for 4 to 8 weeks 4. Cytoprotective drug (misoprostol) for 4 to 8 weeks ____ 8. The next step in treatment when a patient has been on proton pump inhibitors twice daily for 12 weeks and not improving is: 1. Add a prokinetic (metoclopramide) 2. Referral for endoscopy 3. Switch to another proton pump inhibitor 4. Add a cytoprotective drug ____ 9. Infants with reflux are initially treated with: 1. Histamine2 receptor antagonist (ranitidine) 2. Proton pump inhibitor (omeprazole) 3. Anti-reflux maneuvers (elevate head of bed) 4. Prokinetic (metoclopramide) ____ 10. Long-term use of proton pump inhibitors may lead to: 1. Hip fractures in at-risk persons 2. Vitamin B6 deficiency 3. Liver cancer 4. All of the above ____ 11. An acceptable first-line treatment for peptic ulcer disease with positive H. pylori test is: 1. Histamine2 receptor antagonists for 4 to 8 weeks 2. Proton pump inhibitor bid for 12 weeks until healing is complete 3. Proton pump inhibitor bid plus clarithromycin plus amoxicillin for 14 days 4. Proton pump inhibitor bid and levofloxacin for 14 days ____ 12. Treatment failure in patients with peptic ulcer disease associated with H. pylori may be because of: 1. Antimicrobial resistance 2. An ineffective antacid 3. Overuse of proton pump inhibitors 4. All of the above ____ 13. If a patient with H. pylori-positive peptic ulcer disease fails first-line therapy, the second-line treatment is: 1. Proton pump inhibitor bid plus metronidazole plus tetracycline plus bismuth subsalicylate for 14 days 2. Test H. pylori for resistance to common treatment regimens 3. Proton pump inhibitor plus clarithromycin plus amoxicillin for 14 days 4. Proton pump inhibitor and levofloxacin for 14 days ____ 14. After H. pylori treatment is completed, the next step in peptic ulcer disease therapy is: 1. Testing for H. pylori eradication with a serum ELISA test 2. Endoscopy by a specialist 3. A proton pump inhibitor for 8 to 12 weeks until healing is complete 4. All of the above Chapter 39. Hyperlipidemia ____ 1. The overall goal of treating hyperlipidemia is: 1. Maintain an LDL level of less than 160 mg/dL 2. To reduce atherogenesis 3. Lowering apo B, one of the apoliproteins 4. All of the above ____ 2. When considering which cholesterol-lowering drug to prescribe, which factor determines the type and intensity of treatment? 1. Total LDL 2. Fasting HDL 3. Coronary artery disease risk level 4. Fasting total cholesterol ____ 3. First-line therapy for hyperlipidemia is: 1. Statins 2. Niacin 3. Lifestyle changes 4. Bile acid-binding resins ____ 4. James is a 45-year-old patient with an LDL level of 120 and normal triglycerides. Appropriate first-line therapy for James may include diet counseling, increased physical activity, and: 1. A statin 2. Niacin 3. Sterols 4. A fibric acid derivative ____ 5. Joanne is a 60-year-old patient with an LDL of 132 and a family history of coronary artery disease. She has already tried diet changes (increased fiber and plant sterols) to lower her LDL and after 6 months her LDL is slightly higher. The next step in her treatment would be: 1. A statin 2. Niacin 3. Sterols 4. A fibric acid derivative ____ 6. Sharlene is a 65-year-old patient who has been on a lipid-lowering diet and using plant sterol margarine daily for the past 3 months. Her LDL is 135 mg/dL. An appropriate treatment for her would be: 1. A statin 2. Niacin 3. A fibric acid derivative 4. Determined by her risk factors ____ 7. Phil is a 54-year-old male with multiple risk factors who has been on a high-dose statin for 3 months to treat his high LDL level. His LDL is 135 mg/dL and his triglycerides are elevated. A reasonable change in therapy would be to: 1. Discontinue the statin and change to a fibric acid derivative. 2. Discontinue the statin and change to ezetimibe. 3. Continue the statin and add in ezetimibe. 4. Refer him to a specialist in managing patients with recalcitrant hyperlipidemia. ____ 8. Jamie is a 34-year-old pregnant woman with familial hyperlipidemia and elevated LDL levels. What is the appropriate treatment for a pregnant woman? 1. A statin 2. Niacin 3. Fibric acid derivative 4. Bile acid-binding resins ____ 9. Han is a 48-year-old diabetic with hyperlipidemia and high triglycerides. His LDL is 112 mg/dL and he has not tolerated statins. He warrants a trial of a: 1. Sterol 2. Niacin 3. Fibric acid derivative 4. Bile acid-binding resin ____ 10. Jose is a 12-year-old overweight child with a total cholesterol of 180 mg/dL and LDL of 125 mg/dL. Along with diet education and recommending increased physical activity, a treatment plan for Jose would include ____________ with a reevaluation in 6 months. 1. Statins 2. Niacin 3. Sterols 4. Bile acid-binding resins ____ 11. Monitoring of a patient who is on a lipid-lowering drug includes: 1. Fasting total cholesterol every 6 months 2. Lipid profile with attention to serum LDL 6 to 8 weeks after starting therapy, then again in 6 weeks 3. Complete blood count, C-reactive protein, and erythrocyte sedimentation rate after 6 weeks of therapy 4. All of the above ____ 12. Before starting therapy with a statin, the following baseline laboratory values should be evaluated: 1. Complete blood count 2. Liver function (ALT/AST) and creatine kinase 3. C-reactive protein 4. All of the above ____ 13. When starting a patient on a statin, education would include: 1. If they stop the medication their lipid levels will return to pretreatment levels. 2. Medication is a supplement to diet therapy and exercise. 3. If they have any muscle aches or pain, they should contact their provider. 4. All of the above ____ 14. Omega 3 fatty acids are best used to help treat: 1. High HDL 2. Low LDL 3. High triglycerides 4. Any high lipid value ____ 15. When are statins traditionally ordered to be taken? 1. At bedtime 2. At noon 3. At breakfast 4. With the evening meal ____ 16. Which the following persons should not have a statin medication ordered? 1. Someone with 3 first- or second-degree family members with history of muscle issues when started on statins 2. Someone with high lipids, but low BMI 3. Premenopausal woman with recent history of hysterectomy 4. Prediabetic male with known metabolic syndrome ____ 17. Fiber supplements are great options for elderly patients who have the concurrent problem of: 1. End-stage renal failure on fluid restriction 2. Recurrent episodes of diarrhea several times a day 3. Long-term issues of constipation 4. Needing to take multiple medications around the clock every 2 hours ____ 18. What is considered the order of statin strength from lowest effect to highest? 1. Lovastatin, Simvastatin, Rosuvastatin 2. Rosuvastatin, Lovastatin, Atorvastatin 3. Atorvastatin, Rosuvastatin, Simvastatin 4. Simvastatin, Atorvastatin, Lovastatin Chapter 43. Smoking Cessation ____ 1. Nicotine withdrawal symptoms include: 1. Nervousness 2. Increased appetite 3. Difficulty concentrating 4. All of the above ____ 2. If a patient wants to quit smoking, nicotine replacement therapy is recommended if the patient: 1. Smokes more than 10 cigarettes a day 2. Smokes within 30 minutes of awakening in the morning 3. Smokes when drinking alcohol 4. All of the above ____ 3. Instructions for a patient who is starting nicotine replacement therapy include: 1. Smoke less than 10 cigarettes a day when starting nicotine replacement. 2. Nicotine replacement will help with the withdrawal cravings associated with quitting tobacco. 3. Nicotine replacement can be used indefinitely. 4. Nicotine replacement therapy is generally safe for all patients. ____ 4. Nicotine replacement therapy should not be used in which patients? 1. Pregnant women 2. Patients with worsening angina pectoris 3. Patients who have just suffered an acute myocardial infarction 4. All of the above ____ 5. Instructions for the use of nicotine gum include: 1. Chew the gum quickly to get a peak effect. 2. The gum should be “parked” in the buccal space between chewing. 3. Acidic drinks such as coffee help with the absorption of the nicotine. 4. The highest abstinence rates occur if the patient chews the gum when he or she is having cravings. ____ 6. Patients who choose the nicotine lozenge to assist in quitting tobacco should be instructed: 1. Chew the lozenge well. 2. Drink at least 8 ounces of water after the lozenge dissolves. 3. Use one lozenge every 1 to 2 hours (at least nine per day with a maximum of 20 per day). 4. A tingling sensation in the mouth should be reported to the provider. ____ 7. Transdermal nicotine replacement (the patch) is an effective choice in tobacco cessation because: 1. The patch provides a steady level of nicotine without reinforcing oral aspects of smoking. 2. There is the ability to “fine tune” the amount of nicotine that is delivered to the patient at any one time. 3. There is less of a problem with nicotine toxicity than other forms of nicotine replacement. 4. Transdermal nicotine is safer in pregnancy. ____ 8. The most common adverse effect of the transdermal nicotine replacement patch is: 1. Nicotine toxicity 2. Tingling at the site of patch application 3. Skin irritation under the patch site 4. Life-threatening dysrhythmias ____ 9. If a patient is exhibiting signs of nicotine toxicity when using transdermal nicotine, they should remove the patch and: 1. Wash the area thoroughly with soap and water. 2. Flush the area with clear water. 3. Reapply a new patch in 8 hours. 4. Take acetaminophen for the headache associated with toxicity. ____ 10. When a patient is prescribed nicotine nasal spray for tobacco cessation, instructions include: 1. Inhale deeply with each dose to ensure deposition in the lungs. 2. The dose is one to two sprays in each nostril per hour, not to exceed 40 sprays per day. 3. If they have a sensation of “head rush” this indicates the medication is working well. 4. Nicotine spray may be used for up to 12 continuous months. ____ 11. If prescribing bupropion (Zyban) for tobacco cessation, the instructions to the patient include: 1. Bupropion (Zyban) is started 1 to 2 weeks before the quit date. 2. Nicotine replacement products should not be used with bupropion. 3. If they smoke when taking bupropion they may have increased anxiety and insomnia. 4. Because they are not using bupropion as an antidepressant, they do not need to worry about increased suicide ideation when starting therapy. ____ 12. Varenicline (Chantix) may be prescribed for tobacco cessation. Instructions to the patient who is starting varenicline include: 1. The maximum time varenicline can be used is 12 weeks. 2. Nausea is a sign of varenicline toxicity and should be reported to the provider. 3. The starting regimen for varenicline is start taking 1 mg twice a day a week before the quit date. 4. Neuropsychiatric symptoms may occur. ____ 13. The most appropriate smoking cessation prescription for pregnant women is: 1. A nicotine replacement patch at the lowest dose available 2. Bupropion (Zyban) 3. Varenicline (Chantix) 4. Nonpharmacologic measures Chapter 49. Men as Patients ____ 1. The factor that has the greatest effect on males developing male sexual characteristics is: 1. Cultural beliefs 2. Effective male role models 3. Adequate intake of testosterone in the diet 4. Androgen production ____ 2. When assessing a male for hypogonadism prior to prescribing testosterone replacement, serum testosterone levels are drawn: 1. Without regard to time of day 2. First thing in the morning 3. Late afternoon 4. In the evening ____ 3. Some research supports that testosterone replacement therapy may be indicated in which of the following diagnoses in men? 1. Age-related decrease in cognitive functioning 2. Metabolic syndrome 3. Decreased muscle mass in aging men 4. All of the above ____ 4. The goal of testosterone replacement therapy is: 1. Absence of all hypogonadism symptoms 2. Testosterone levels in the mid-normal range 1 week after an injection 3. Testosterone levels in the mid-normal range just prior to the next injection 4. Avoidance of high serum testosterone levels during therapy ____ 5. While on testosterone replacement, hemoglobin and hematocrit levels should be monitored. Levels suggestive of excessive erythrocytosis or abuse are: 1. Hemoglobin 14 g/dl or hematocrit 39% 2. Hemoglobin 11.5 g/dl or hematocrit 31% 3. Hemoglobin 13 g/dl or hematocrit 38% 4. Hemoglobin 17.5 g/dl or hematocrit 54% ____ 6. Monitoring of an older male patient on testosterone replacement includes: 1. Oxygen saturation levels at every visit 2. Serum cholesterol and lipid profile every 3 to 6 months 3. Digital rectal prostate screening exam at 3 and 6 months after starting therapy 4. Bone mineral density at 3 months and 6 months after starting therapy ____ 7. When prescribing phosphodiesterase type 5 (PDE-5) inhibitors such as sildenafil (Viagra) patients should be screened for use of: 1. Statins 2. Nitrates 3. Insulin 4. Opioids ____ 8. Men who are prescribed phosphodiesterase type 5 (PDE-5) inhibitors for erectile dysfunction should be educated regarding the adverse effects of the drug which include: 1. Hearing loss 2. Hypotension 3. Delayed ejaculation 4. Dizziness ____ 9. Male patients who should not be prescribed phosphodiesterase type 5 (PDE-5) inhibitors include: 1. Diabetics 2. Those who have had an acute myocardial infarction in the past 6 months 3. Patients who are deaf 4. Patients under age 60 years of age ____ 10. Monitoring of male patients who are using phosphodiesterase type 5 (PDE-5) inhibitors includes: 1. Serum fasting glucose levels 2. Cholesterol and lipid levels 3. Blood pressure 4. Complete blood count Chapter 50. Children as Patients ____ 1. The Pediatric Research Equity Acts requires: 1. All children be provided equal access to drug research trials 2. Children to be included in the planning phase of new drug development 3. That pediatric drug trials guarantee children of multiple ethnic groups are included 4. All applications for new active ingredients, new indications, new dosage forms, or new routes of administration require pediatric studies ____ 2. The Best Pharmaceuticals for Children Act: 1. Includes a pediatric exclusivity rule which extends the patent on drugs studied in children 2. Establishes a committee that writes guidelines for pediatric prescribing 3. Provides funding for new drug development aimed at children 4. Encourages manufacturers specifically to develop pediatric formulations ____ 3. The developmental variation in Phase I enzymes has what impact on pediatric prescribing? 1. None, Phase I enzymes are stable throughout childhood. 2. Children should always be prescribed lower than adult doses per weight due to low enzyme activity until puberty. 3. Children should always be prescribed higher than adult doses per weight due to high enzyme activity. 4. Prescribing dosages will vary based on the developmental activity of each enzyme, at times requiring lower than adult doses and other times higher than adult doses based on the age of the child. ____ 4. Developmental variation in renal function has what impact on prescribing for infants and children? 1. Lower doses of renally excreted drugs may be prescribed to infants younger than age 6 months. 2. Higher doses of water soluble drugs may need to be prescribed because of increased renal excretion. 3. Renal excretion rates have no impact on prescribing. 4. Parents need to be instructed on whether drugs are renally excreted or not. ____ 5. Topical corticosteroids are prescribed cautiously in young children because: 1. They may cause an intense hypersensitivity reaction 2. Of hypothalamic-pituitary-adrenal axis suppression 3. Corticosteroids are less effective in young children 4. Young children may accumulate corticosteroids, leading to toxic levels ____ 6. Liza is breastfeeding her 2-month-old son and has an infection that requires an antibiotic. What drug factors influence the effect of the drug on the infant? 1. Maternal drug levels 2. Half-life 3. Lipid-solubility 4. All of the above ____ 7. Drugs that are absolutely contraindicated in lactating women include: 1. Selective serotonin reuptake inhibitors 2. Antiepileptic drugs such as carbamazepine 3. Antineoplastic drugs such as methotrexate 4. All of the above ____ 8. Zia is a 4-month-old patient with otitis media. Education of his parents regarding administering oral antibiotics to an infant includes: 1. How to administer an oral drug using a medication syringe 2. Mixing the medication with a couple of ounces of formula and putting it in a bottle 3. Discontinuing the antibiotic if diarrhea occurs 4. Calling for an antibiotic change if the infant chokes and sputters during administration ____ 9. To increase adherence in pediatric patients a prescription medication should: 1. Have a short half-life 2. Be the best tasting of the effective drugs 3. Be the least concentrated form of the medication 4. Be administered 3 or 4 times a day ____ 10. Janie is a 5-month-old breastfed infant with a fever. Treatment for her fever may include: 1. “Baby” aspirin 2. Acetaminophen suppository 3. Ibuprofen suppository 4. Alternating acetaminophen and ibuprofen Chapter 51. Geriatric Patients ____ 1. Principles of prescribing for older adults include: 1. Avoiding prescribing any newer high-cost medications 2. Starting at a low dose and increasing the dose slowly 3. Keeping the total dose at a lower therapeutic range 4. All of the above ____ 2. Sadie is a 90-year-old patient who requires a new prescription. What changes in drug distribution with aging would influence prescribing for Sadie? 1. Increased volume of distribution 2. Decreased lipid solubility 3. Decreased plasma proteins 4. Increased muscle-to-fat ratio ____ 3. Glen is an 82-year-old patient who needs to be prescribed a new drug. What changes in elimination should be taken into consideration when prescribing for Glen? 1. Increased glomerular filtration rate (GFR) will require higher doses of some renally excreted drugs. 2. Decreased tubular secretion of medication will require dosage adjustments. 3. Thin skin will cause increased elimination via sweat. 4. Decreased lung capacity will lead to measurable decreases in lung excretion of drugs. ____ 4. A medication review of an elderly person’s medications involves: 1. Asking the patient to bring a list of current prescription medications to the visit 2. Having the patient bring all of their prescription, over-the-counter, and herbal medications to the visit 3. Asking what other providers are writing prescriptions for them 4. All of the above ____ 5. Steps to avoid polypharmacy include: 1. Prescribing two or fewer drugs from each drug class 2. Reviewing a complete drug history every 12 to 18 months 3. Encouraging the elderly patient to coordinate their care with all of their providers 4. Evaluating for duplications in drug therapy and discontinuing any duplications ____ 6. Robert is a 72-year-old patient who has hypertension and angina. He is at risk for common medication practices seen in the elderly including: 1. Use of another person’s medications 2. Hoarding medications 3. Changing his medication regimen without telling his provider 4. All of the above ____ 7. To improve positive outcomes when prescribing for the elderly the nurse practitioner should: 1. Assess cognitive functioning in the elder 2. Encourage the patient to take a weekly “drug holiday” to keep drug costs down 3. Encourage the patient to cut drugs in half with a knife to lower costs 4. All of the above ____ 8. When an elderly diabetic patient is constipated the best treatment options include: 1. Mineral oil 2. Bulk-forming laxatives such as psyllium 3. Stimulant laxatives such as senna 4. Stool softeners such as docusate ____ 9. Delta is an 88-year-old patient who has mild low-back pain. What guidelines should be followed when prescribing pain management for Delta? 1. Keep the dose of oxycodone low to prevent development of tolerance. 2. Acetaminophen is the first-line drug of choice. 3. Avoid prescribing NSAIDs. 4. Add in a short-acting benzodiazepine for a synergistic effect on pain. ____ 10. Robert is complaining of poor sleep. Medications that may contribute to sleep problems in the elderly include: 1. Diuretics 2. Trazodone 3. Clonazepam 4. Levodopa ____ 11. The GFRs for a 91-year-old woman who weighs 93 pounds and is 5'1" with a serum creatinine of 1.1, and for a 202-pound, 25-year-old male who is 5'9" with the same serum creatinine according to the Cockcroft Gault formula are: 1. 25ml/ min and 133 mL/min respectively 2. 25 mL/min and 103 mL/min respectively 3. 22 ml/min and 133 mL/min respectively 4. 22 ml/min and 103 mL/min respectively ____ 12. In geriatric patients, the percentage of body fat is increased. What are the pharmacologic implications of this physiologic change? 1. A lipid-soluble medication will be eliminated more quickly and not work as well. 2. A lipid-soluble medication will accumulate in fat tissue and its duration of action may be prolonged. 3. Absorption of lipid-soluble drugs is impaired in older adults. 4. The bioavailability of the lipid-soluble drug will be increased in older adults. ____ 13. All of the following statements about the Beer’s List are true except: 1. It is a list of medications or medication classes that should generally be avoided in persons 65 years or older because they are either ineffective or they pose unnecessarily high risk for older persons and a safer alternative is available. 2. It is derived from the expert opinion of one geriatrician and is not evidence-based. 3. These criteria have been adopted by the Centers for Medicare and Medicaid Services for regulation of long-term care facilities. 4. These criteria are directed at the general population of patients over 65 years of age and do not take disease states into consideration. ____ 14. You are reviewing the data from several meta-analyses that addressed the most common causes of adverse drug reactions in the older adult. Which of the following would you find to be decreased and the most common cause of these problems in older adults? 1. Body fat content 2. Liver function 3. Renal function/clearance 4. Plasma albumin levels ____ 15. Which of the following is not consistent with the rules for geriatric prescribing: 1. Half-life will be longer in older adults 2. Steady state is reached more quickly in the older adult 3. Reduce the number of drugs in the patient's regimen whenever possible 4. Adverse drug responses present atypically in the older adult [Show More]

Last updated: 1 year ago

Preview 1 out of 161 pages

Reviews( 0 )

$10.00

Add to cart

Instant download

Can't find what you want? Try our AI powered Search

OR

GET ASSIGNMENT HELP
90
0

Document information


Connected school, study & course


About the document


Uploaded On

Nov 15, 2020

Number of pages

161

Written in

Seller


seller-icon
quiz_bit

Member since 3 years

552 Documents Sold


Additional information

This document has been written for:

Uploaded

Nov 15, 2020

Downloads

 0

Views

 90

Document Keyword Tags

Recommended For You


$10.00
What is Browsegrades

In Browsegrades, a student can earn by offering help to other student. Students can help other students with materials by upploading their notes and earn money.

We are here to help

We're available through e-mail, Twitter, Facebook, and live chat.
 FAQ
 Questions? Leave a message!

Follow us on
 Twitter

Copyright © Browsegrades · High quality services·